I Refute it Thus!
@Banno pointed me to a recent OP by British philosopher Raymond Tallis, on the Philosophy Now site (thanks!) commenting on Johnson's 'refutation from the stone', to which this famous exclamation is a reference. It is Samuel Johnson's supposed refutation of Bishop George Berkeley's idealist (or immaterialist) philosophy. (The OP appears to be not paywalled (although there's a limit to free reads on their site) so readers can peruse it at their leisure. But as I want to offer a critique of Tallis' critique it is probably advisable to have a look at it first.)
[quote=James Boswell, The Life of Samuel Johnson] After we came out of the church, we stood talking for some time together of Bishop Berkeley's ingenious sophistry to prove the non-existence of matter, and that every thing in the universe is merely ideal. I observed, that though we are satisfied his doctrine is not true, it is impossible to refute it. I never shall forget the alacrity with which Johnson answered, striking his foot with mighty force against a large stone, till he rebounded from it, "I refute it thus!"[/quote]
This is nowadays taught as an example of an informal fallacy ('argumentum ad lapidem'), on the basis that it does not satisfactorily come to terms with the theory it is intended to refute, but rather merely amounts to an assertion or a claim that the argument it opposes is obviously absurd.
In order to assess this, it is necessary to recall what Johnson was so indignant about. Bishop George Berkeley (1685-1753) is known in the history of philosophy as an empiricist, but also as an idealist. Coining the famous aphorism 'esse est percipe', Berkeley denied that matter was an independently-existing substance (in the philosophical sense, as the bearer of attributes.) It may seem puzzling to be both empiricist and idealist, but Berkeley had seized upon the principle articulated by other empiricists, Locke and Hume, that all knowledge is acquired by way of sensory impressions.
In Berkeley's philosophy, 'there are only two kinds of things: spirits and ideas. Spirits are simple, active beings which produce and perceive ideas; ideas are passive beings which are produced and perceived' [sup] 1[/sup]. His concept of 'spirit' seems something like the abstract form of 'the conscious subject' or 'observing mind' (bearing in mind that for Berkeley, the mind of God was an omnipresent observer), and his concept of 'idea' is close to the concept of 'sense-object' or 'phenomenal object'.
It's important to note that while Berkeley denied the existence of matter as a metaphysical substance - again, recalling the philosophical, as distinct from everyday, use of that term - he did not deny the existence of physical objects such as apples or mountains, saying "I do not argue against the existence of any one thing that we can apprehend, either by sense or reflection. That the things I see with mine eyes and touch with my hands do exist, really exist, I make not the least question. The only thing whose existence we deny, is that which philosophers call matter or corporeal substance. And in doing of this, there is no damage done to the rest of mankind, who, I dare say, will never miss it." (Principles #35). Berkeleys idealism is often misinterpreted as denying the reality of objects, but his point is that their reality consists in being perceived, either by finite minds or the divine mind, ensuring coherence and continuity in the world.[sup]2[/sup]
On reading Tallis' analysis, my initial reaction was that it is based on a rather shallow interpretation of what Berkeley intends by 'perception'. I think one could respond to Tallis by saying that idealism understands that cognition comprises many layers, not all of which are conscious or the same as my perception of an object (although admittedly, this might not be something Berkeley makes explicit.) For Berkeley, perception encompasses the whole experience of the world as presented to the mind. In this sense, perception is not just the reception of sensory data but relies on a divinely mediated intelligibility that underpins the coherence of the experienced world.
Tallis gives the example of the lymphatic system, which we, as subjects, are unaware of, but without which we would surely perish. The same can be said for many para-sympathetic and unconscious metabolic and cognitive processes that underpin our cognitive abilities. He points out that these are not available to conscious perception and raises the question of how they continue to exist "unperceived," as it were. All well and goodbut need they be consciously perceived in Berkeleys view? Tallis focus on the layers of material complexity underpinning Johnsons body and the composition of the stone presumes that the existence of these unperceived levels reinforces the independence of material objects and organic processes from perception. However, each of these layersthe lymphatic and cognitive systems, on the one side, and the stones chemical composition and physical characteristics, on the otherare only known to us through conceptual and perceptual frameworks, including scientific theories, which we bring to bear in understanding them. In other words, these layers are not directly available in the act of perceiving the stone, but if we include them in our consideration of the act of knowing, they are themselves presented to us as what Berkeley would call "ideas."
Likewise, when we analyze the stone, breaking it down into its molecular structure, atomic composition, or even quantum fields, what we uncover are not "things-in-themselves" ( a material substance in Berkeley's terms) but rather representations within our scientific and perceptual models. For Berkeley, what we encounter are representations all the way downnot of an independent material substance, but of ideas structured and sustained by mind. The layers of analysis Tallis invokes ultimately reinforce this view rather than refuting it, as such representations are deeply tied to our cognitive capacities, conceptual schemas, and observational tools (This level of analysis is admittedly more sophisticated than anything Berkeley offered, but it is still consonant with his overall philosophy, although made much more explicit in the later forms of idealism such as Kant's and Schopenhauer's). Similarly, when we analyze the body's systems, we dont directly access 'the body as it is' but only the intelligible structure as it appears through layers of interpretation. In this way, both the stone and Johnsons body remain phenomena structured and experienced within the bounds of perception and thought.
Elsewhere, Raymond Tallis is a staunch critic of philosophical materialism, for instance in his books such as Aping Mankind. While he is not defending materialism here, his depiction of idealism seems to fall back on assumptions about material reality that he critiques in other contexts.
So does Tallis argument effectively challenge Berkeleys idealism, or does it reveal a deeper reliance on the very conceptual frameworks that idealism critiques? And how might more recent developments in philosophy and science, such as quantum mechanics and cognitive science, have bearing on this debate?
-----
[sup]1[/sup] Bettcher T. M: Berkeley: A Guide for the Perplexed, Continuum Publishing, 2008. p. 14.
[sup]2[/sup] There are good updated translations of Berkeley's writings presented on the Early Modern Texts website.
[quote=James Boswell, The Life of Samuel Johnson] After we came out of the church, we stood talking for some time together of Bishop Berkeley's ingenious sophistry to prove the non-existence of matter, and that every thing in the universe is merely ideal. I observed, that though we are satisfied his doctrine is not true, it is impossible to refute it. I never shall forget the alacrity with which Johnson answered, striking his foot with mighty force against a large stone, till he rebounded from it, "I refute it thus!"[/quote]
This is nowadays taught as an example of an informal fallacy ('argumentum ad lapidem'), on the basis that it does not satisfactorily come to terms with the theory it is intended to refute, but rather merely amounts to an assertion or a claim that the argument it opposes is obviously absurd.
In order to assess this, it is necessary to recall what Johnson was so indignant about. Bishop George Berkeley (1685-1753) is known in the history of philosophy as an empiricist, but also as an idealist. Coining the famous aphorism 'esse est percipe', Berkeley denied that matter was an independently-existing substance (in the philosophical sense, as the bearer of attributes.) It may seem puzzling to be both empiricist and idealist, but Berkeley had seized upon the principle articulated by other empiricists, Locke and Hume, that all knowledge is acquired by way of sensory impressions.
In Berkeley's philosophy, 'there are only two kinds of things: spirits and ideas. Spirits are simple, active beings which produce and perceive ideas; ideas are passive beings which are produced and perceived' [sup] 1[/sup]. His concept of 'spirit' seems something like the abstract form of 'the conscious subject' or 'observing mind' (bearing in mind that for Berkeley, the mind of God was an omnipresent observer), and his concept of 'idea' is close to the concept of 'sense-object' or 'phenomenal object'.
It's important to note that while Berkeley denied the existence of matter as a metaphysical substance - again, recalling the philosophical, as distinct from everyday, use of that term - he did not deny the existence of physical objects such as apples or mountains, saying "I do not argue against the existence of any one thing that we can apprehend, either by sense or reflection. That the things I see with mine eyes and touch with my hands do exist, really exist, I make not the least question. The only thing whose existence we deny, is that which philosophers call matter or corporeal substance. And in doing of this, there is no damage done to the rest of mankind, who, I dare say, will never miss it." (Principles #35). Berkeleys idealism is often misinterpreted as denying the reality of objects, but his point is that their reality consists in being perceived, either by finite minds or the divine mind, ensuring coherence and continuity in the world.[sup]2[/sup]
On reading Tallis' analysis, my initial reaction was that it is based on a rather shallow interpretation of what Berkeley intends by 'perception'. I think one could respond to Tallis by saying that idealism understands that cognition comprises many layers, not all of which are conscious or the same as my perception of an object (although admittedly, this might not be something Berkeley makes explicit.) For Berkeley, perception encompasses the whole experience of the world as presented to the mind. In this sense, perception is not just the reception of sensory data but relies on a divinely mediated intelligibility that underpins the coherence of the experienced world.
Tallis gives the example of the lymphatic system, which we, as subjects, are unaware of, but without which we would surely perish. The same can be said for many para-sympathetic and unconscious metabolic and cognitive processes that underpin our cognitive abilities. He points out that these are not available to conscious perception and raises the question of how they continue to exist "unperceived," as it were. All well and goodbut need they be consciously perceived in Berkeleys view? Tallis focus on the layers of material complexity underpinning Johnsons body and the composition of the stone presumes that the existence of these unperceived levels reinforces the independence of material objects and organic processes from perception. However, each of these layersthe lymphatic and cognitive systems, on the one side, and the stones chemical composition and physical characteristics, on the otherare only known to us through conceptual and perceptual frameworks, including scientific theories, which we bring to bear in understanding them. In other words, these layers are not directly available in the act of perceiving the stone, but if we include them in our consideration of the act of knowing, they are themselves presented to us as what Berkeley would call "ideas."
Likewise, when we analyze the stone, breaking it down into its molecular structure, atomic composition, or even quantum fields, what we uncover are not "things-in-themselves" ( a material substance in Berkeley's terms) but rather representations within our scientific and perceptual models. For Berkeley, what we encounter are representations all the way downnot of an independent material substance, but of ideas structured and sustained by mind. The layers of analysis Tallis invokes ultimately reinforce this view rather than refuting it, as such representations are deeply tied to our cognitive capacities, conceptual schemas, and observational tools (This level of analysis is admittedly more sophisticated than anything Berkeley offered, but it is still consonant with his overall philosophy, although made much more explicit in the later forms of idealism such as Kant's and Schopenhauer's). Similarly, when we analyze the body's systems, we dont directly access 'the body as it is' but only the intelligible structure as it appears through layers of interpretation. In this way, both the stone and Johnsons body remain phenomena structured and experienced within the bounds of perception and thought.
Elsewhere, Raymond Tallis is a staunch critic of philosophical materialism, for instance in his books such as Aping Mankind. While he is not defending materialism here, his depiction of idealism seems to fall back on assumptions about material reality that he critiques in other contexts.
So does Tallis argument effectively challenge Berkeleys idealism, or does it reveal a deeper reliance on the very conceptual frameworks that idealism critiques? And how might more recent developments in philosophy and science, such as quantum mechanics and cognitive science, have bearing on this debate?
-----
[sup]1[/sup] Bettcher T. M: Berkeley: A Guide for the Perplexed, Continuum Publishing, 2008. p. 14.
[sup]2[/sup] There are good updated translations of Berkeley's writings presented on the Early Modern Texts website.
Comments (171)
I have never heard it taught that way. It seems like an erroneous reading. Here is what Wikipedia says:
Quoting Wikipedia
I don't think it's hard to see that Johnson is not doing this. He is not saying, "You're wrong because you're wrong." In fact he is giving an argument, not begging the question. What you are accusing him of is ignoratio elenchus, not begging the question.
What is his argument?
Note that this is perfectly valid. You are merely disputing premise (1).
Further, how does one dispute premise (1) without themselves begging the question? They must explicate Berkeley's theory at least to the extent that a rejection of (1) is understood not to be an ad hoc rejection. That is, the listener has to come away saying, "Ah, I see what Berkeley was saying, why he was saying it, and why it does not entail (1)." That's basically the question: supposing Johnson's first premise is false, then what does follow from Berkeley's "idealism"?
(I've read the article, but I want to revisit Berkeley before I would comment on it.)
Johnson's exclamation is the historical origin of the expression 'argumentum ad lapidem'.
Quoting Leontiskos
But it's equally the case that Johnson misunderstands Berkeley. Johnson is intending to demonstrate that Berkeley's argument entails that the stone does not really exist, but Berkeley doesn't make such a claim. Berkeley himself acknowledges that 'I do not argue against the existence of any one thing that we can apprehend, either by sense or reflection. That the things I see with mine eyes and touch with my hands do exist, really exist, I make not the least question.' His argument is not that stones, and feet, do not exist, but that there is no material substance apart from and separate to the manifold impressions that the stone makes on our sensory organs (including the sense of touch).
The composition and nature of the stone is a matter for physical chemistry and physics. And it is nowadays well known that minute analysis of the stone reveals ever-smaller components or particles from which it is composed, until the sub-atomic level is reached, at which point the nature of the so-called components of matter, if that is what 'material substance' is supposed to comprise, becomes quite ambiguous. In fact modern sub-atomic physics has not done much to support the kind of 'argument' that Johnson is proposing.
And therefore:
Quoting Leontiskos
I clarify this because you have accused others (including myself) of the so-called "argumentum ad lapidem," and so it is worth recognizing that it is not the informal fallacy of begging the question (which is what the Wikipedia writer is really claiming). Both are informal fallacies, but one requires a response/clarification and one does not.
Ignoratio Elenchi
Quoting Wayfarer
If Wikipedia says that Johnson was begging the question, then I say that Wikipedia is wrong. It happens.
Also I think there are grounds on which Berkeley can be criticized, I certainly dont regard him as having the last word on the subject. Im specifically taking issue with Tallis argument.
Well <here> is the accusation I had in mind, and I pointed out the same error in two subsequent posts, here and here.
I think it is important to understand that Johnson is not begging the question, but the point is made and I'll leave it there.
Quoting Wayfarer
I didn't mean it an accusation, but as counter-argument. Anyway, I revisited your objections in this more recent post (and the one immediately after with the supporting quote from Schopenhauer.)
Quoting Wayfarer
Conscious or unconscious I don't think is relevant as long as they are held within a mind. I think Berkeley is clear about that. It doesn't have to be perceived by us per say, but something like God. Thus humanity could be completely ignorant of any science and it still exist.
Quoting Wayfarer
This is a fantastic analysis of knowledge vs truth. But is Berkeley really saying that? Perhaps Berkeley flirted with this a bit like so many do, but ultimately didn't fully realize it as he had to rely on God for his philosophy to work. This is of course based on the information presented here and in the article. If you can find a part of Berkeley's work where he doesn't ultimately rely on God when a human cannot perceive what exists, feel free to post it.
I think the issue is that premise #1 is wrong, false for the reason explained by Wayfarer. Since that premise is false, and obviously just intentionally designed to produce the conclusion desired, it is a matter of begging the question.
Quoting Philosophim
I think this is the real issue. "Matter" is a concept developed by Aristotle to account for the observed temporal continuity of bodies, objects, which are actively changing. Both Hume and Newton take this temporal continuity for granted, as "inertia", but they assign it to the temporal extension of the activity, the motion, rather than assigning it to a substantial body or object, as Aristotle did.
As Berkely shows, the concept of "matter" is unnecessary, so the assumption of an underlying continuous body, or object is likewise unnecessary. Furthermore, it is impossible that "matter" by its proper conception, has properties, because properties are formal. This allows that whatever is referred to by "inertia", cannot be a property of matter, but it can effectively replace the concept of "matter", to account for the observed temporal continuity. As a result, temporal continuity is no longer restricted to bodies or objects, but is allowed to be a feature of motion itself. Motion itself is "the object", with the property of temporal continuity, and there is no need to assume a body which is moving, thereby enabling the concept of "energy" as an existent thing.
I think it's at least reasonable. He seems to be saying that we know matter exists in the first place because of our experience of our body, and it is the interaction of Johnson's material body with the object of the rock that secures the conclusion that the rock is material. I said something similar to Tallis' premise recently:
Quoting Leontiskos
Experience of our bodies shows us that there are existing things which are important and yet are not perceived:
Quoting Tallis
And if Berkeley says that God constantly perceives Tallis' lymphatic system, Tallis might ask whether that sort of reliance on God constitutes a falsifiable claim.
Most of this seems to depend on just what definition of 'matter' we place in Berkeley's mouth.
I don't grant your imputation of specious motive.
I think it's extremely hard to fathom the sense in which God exists. I don't want to drag the thread into the direction of theology, except to say that God does not exist in the same sense that objects do.
[hide="Reveal"](In neoplatonic philosophy, which provided the philosophical framework for later theology, the ground of existence is not itself something that exists. The ultimate source or ground of being transcends existence, beyond coming to be and passing away, prior to and more fundamental than discrete existential categories subjects. This is expressed by the Neoplatonic "One", the ineffable, unqualified source from which all existence emanates, but which cannot be directly characterized as something that exists. See 'God does not Exist'. )
Quoting Philosophim
Fair point! As I acknowledged in the OP:
Quoting Wayfarer
But, now you mention it, and as I've brought Berkeley up, I will spend some time perusing the excellent translations in Early Modern Texts to see if I can find more support for my argument (although not today, regrettably, domestic duties call.)
[/hide]Quoting Leontiskos
You should know better than to confuse the metaphysical with the empirical. The point of the principle of falsifiability was to be able to distinguish metaphysical from empirical claims, but it does not aim to falsify metaphysics. In other words, a metaphysical posit is not challenged by its not being falsifiable.
Quoting Leontiskos
Berkeley's definition is material substance, but 'substance' here in the philosophical sense of 'the bearer of attributes', not a 'particular kind of matter with uniform properties' (which is the usual meaning). Berkeley rejected the commonsense notion of matter as an independently existing substance with intrinsic properties, instead saying that what we consider to be matter is in reality a collection or an aggregate of sensory experiences and perceptions. Objects are aggregates of sensible qualities like color, shape, hardness, and texture, which only exist when they are being perceived by an observer. His system fills in the obvious lacuna in that account by positing the Divine Intellect as a universal observer.
Youre right that much of our own minds and bodies remains unperceived from our subjective perspective. But when we turn our attention to these unperceived attributeswhether mental or physicalthey are thereby brought into the realm of perception and cognition. I think this points to a broader idealist insight (perhaps more up-to-date than Berkeley himself articulated): that what we know of the world is always mediated by mind, whether its directly or only potentially perceived. While Berkeleys account may not fully address these layers of cognition, his core claimthat existence is tied to being perceived or perceivableremains cogent.
(That's all for today I'm afd till tomorrow, but thank you for your comments.)
No, I don't think that's right at all. The same principle holds in metaphysics, or more generally, in argument of any kind. It's called the principle of sufficient reason. If there is no possibility of refuting Berkeley's claim, then it transgresses the principle of sufficient reason (for in that case Berkeley has no reason to assert it). Falsifiability is not limited to empirical matters; it's just that in non-empirical matters the falsification takes a non-empirical form.
Quoting Wayfarer
Sure, but Tallis' point stands. The lymphatic system continues to sustain us, perceived or unperceived. Tallis would not disagree that perceptible objects can be perceived at one time and unperceived at another.
The reason Johnson's argument is curious at all is because Berkeley's claim that matter does not exist is so implausible. Or in other words: Berkeley has the burden of proof. If he didn't, then Johnson's action would not attract as much attention as it has.
I really don't know Berkeley well enough to say how he would interact with Tallis, but Tallis' paper is somewhat interesting.
What's kind of surprising in many of these conversations is that people don't bother to spell out what they mean when they say "materialism", "immaterialism", "Idealism", "mentalism", "neutral monism", etc.
He simply assumes people will know what he means. I can only surmise that he thinks that "materialism" implies solidity. Why is this so self-evident given how much more sophisticated out understanding of matter is.
And as others have pointed out, that something is solid does not imply anything about the ontological status of the object.
Why not? The motive of refutation is obvious, even explicit, "I refute it thus". And the premise you stated "1. If Berkeley were right, *this* would never happen." is arguably false, and clearly designed for the purpose of that refutation. It looks like a very clear cut example of begging the question to me.
The conclusion "this act refutes Berkeley" is only derived if the very dubious premise (this act will refute Berkeley), which is designed specifically for that purpose, of refuting Berkeley, is accepted. One could not imagine a better example of begging the question.
Every premise is designed for the purpose of the conclusion, and every premise of a refutation is designed for the purpose of the refutation. Perhaps you are the one begging the question, here.
Quoting Metaphysician Undercover
That's how arguments work. You design premises to reach a conclusion. This whole thread is a testament to the fact that it is not dubious, for the precise point of Tallis' paper is to show that it is not even an ignoratio elenchus, much less a begging of the question. "In what follows, I want to persuade you that the stone-kicking may deliver more than may be apparent at first sight."
Completely different to the principle of falsifiability, youre shifting the goal posts.
Quoting Leontiskos
But you're misinterpreting Berkeley in exactly the same way Johnson did, which is why Johnson's response was a fallacy. Berkeley does not deny the existence of objects - that has already been shown. Kick a rock, and it hurts your foot. Picked up and thrown, a stone will break a window. No contest! What Berkeley is denying is the reality of a material substance, of 'matter' as an abstraction that is separate from the experiential qualities of objects.
For Berkeley, the reality of objects consists entirely in their experiential qualitieswhat is seen, touched, or otherwise perceived. The notion of 'matter' as an abstraction existing independently of those qualities adds nothing to our understanding and, in his view, is incoherent.
[quote=I Refute Him Thus! Misunderstanding Berkeley;https://www.irishphilosophy.com/2016/03/12/berkeleys-immaterialism/]When we perceive an object what is it for us? Berkeley suggests that it a collection of all the ideas (perceptions) conveyed to us by our senses. Take an apple: a certain colour, taste, smell, figure and consistence having been observed to go together, are accounted one distinct thing, signified by the name 'apple'. If you take away those ideas given by the senses, there is nothing left of the apple, not even its solidity nor the space it takes up.
Given this, why presume there is some external substance that is causing perceptions? If we lose this concept, how does that affect what really exists? It doesnt affect it at all, says Berkeley: The philosophers lose their abstract or unperceived Matter Pray what do the rest of mankind lose? As for bodies, etc, we have them still
Instead, Berkeley argues that our talk of existence is purely talk of ideas, or potential ideas: The table I write on I say exists; that is, I see and feel it: and if I were out of my study I should say it existed; meaning thereby that if I was in my study I might perceive it, or that some other spirit (or mind) actually does perceive it. And from this Berkeley argues that the idea of an sensible object that cannot be perceived is incoherent it is essential to it that it must be possible to perceive it. Thus, all sensible objects are necessarily dependant on minds. [/quote]
The Meaning of 'Substance' in Berkeley
As mentioned in the original post, here we're dealing with claims about substance in the philosophical, not everyday, sense. This is derived from Aristotle's metaphysics, via the Latin translation of the Greek 'ousia'. But Aristotle doesnt treat matter (hyle) as a substance in its own right. For Aristotle, matter is pure potentiality (dynamis) a potential something that is not yet actualized. On its own, matter is indeterminate and without form. Form (eidos) is what actualizes matter, giving it structure, purpose, and identity. It is through form that matter becomes a substancea unified entity that can exist and be identified. So the idea of a 'material substance' would be equally incompatible with Aristotelian metaphysics (and this is so, even while acknowledging that Berkeley, with his rejection of universals, was far from Aristotelian.)
In essence, Berkeleys rejection of material substance is a critique of the early modern philosophers (specifically Descartes and Locke) who inherited and transformed Aristotelian metaphysics into the notion of a "material substratum." For Locke, material substance was posited as the "unknown support" underlying sensible qualities, but something inherently beyond perception, and of which we only receive impressions (the basis of Locke's representative realism). For Descartes, matter was res extensa, entirely lacking in intelligence and possessing only spatial extension, all of the functions of intelligence residing in res cogitans, the so-called 'thinking substance'.
Berkeley attacks these ideas, arguing that a material substance, as conceived by the early moderns, is a metaphysical fiction. We never perceive it directly, nor do we have any coherent idea of what it is. Instead of positing a mysterious "substratum," Berkeley simplifies the metaphysics: reality consists of ideas in minds, and there is no need for an independent material "substance." This is what Berkeley sees as incoherent. And I believe it is!
Of course, not being a clergyman or bound by theological traditions, Kastrup has no need for a personal god. Mind-at-Large lacks intentionality, isn't a personal being, and doesnt function as a source of morality or any of the other theological elements one might associate with divinity.
Just out of interest, do you interpret Carl Jung as an idealist in the way Kastrup does? It seems like it could offer a better explanation of the collective unconscious and shared human symbolism; something they never really clarified when I studied Jung.
As for Kastrup's 'mind at large;' I wrote an essay on Medium about that, although it's unlisted as I'm not entirely happy with it. The salient point that I was concerned with, was the inevitable tendency to objectify the 'mind at large' as some kind of really existing entity, which is a fatal mistake in this matter. It is the tendency towards reification (thing-ifying) which is the most insuperable problem for the modern mindset. The antidote to it can only be the 'way of negation' and of unknowing: we don't and can't see the mind, because we are it, it is never something other to us. (I think this is something which is fundamental in Heidegger, although I'm not a Heidegger scholar.)
Quoting Tom Storm
I don't know if he would agree with 'it doesn't function as a source of morality'. If you look at his books More than Allegory and Brief Peaks Beyond, as well as his online discussions with Swami Sarvapriyananda from the NY Vedanta Centre, they're suffused with references to mystical spirituality.[/hide]
This is how he differentiates himself from Berkeley - and it's highly ingenious. It's from his blog dated August 2015.
In other words, we are all dissociated alters of one great mind. We are all expressions of The One - a familiar spiritual axiom.
Berkeley is in some ways a naive idealist, but then, he was also the first in the Western world to consciously articulate such a philosophy. Ive argued before that theres an historical reason for that: the whole notion of material bodies as being independently real was never really considered before the modern period. Creatures (meaning anything created) are mere nothings, said Meister Eckhardt. And that was because they dont contain their own formative principle, which is bestowed by the Creator. So Im of the view that the reason idealism starts to appear in this period, was precisely because of the trend towards naturalism, and the idea that the material world possesses its own, independent reality, whereas for the pre-moderns, the world was an expression of the divine will. I'm not *advocating* that view or saying it can be restored, but recognising it as 'meta-philosophical' factor in the discussion.
You are neglecting a key point, the need to have truthful premises, in order for the conclusion to be sound. The desire for truth of the premises must take priority over the desire to produce a specific conclusion, or else the conclusion will be unsound, having premises designed just to reach that conclusion with complete disregard for truth. Such an argument would be pointless.
The premise in question appears to be false, and designed solely for the purpose of producing the desired conclusion, in complete disregard for truth or falsity. Therefore it constitutes "assuming the conclusion", begging the question.
In honest logical procedure, in contrast with sophistical rhetoric, the premises are designed to represent the truth. And truth of the premises clearly must be given higher priority in the design of the premises, than the desire to produce a specific conclusion, or else the result will be unsound conclusions and in the worst case, begging the question. Designing your premise for the purpose of producing a specific conclusion with disregard for the truth or falsity of the premise is called "assuming the conclusion" or "begging the question": https://en.wikipedia.org/wiki/Begging_the_question
Quoting Leontiskos
I don't see any point in arguing with you then. You'll just design your premises to produce your desired conclusion, with complete disregard for whether or not the premises are true. What's the point in debating anything with a person who's goal is to produce premises which will logically support a prejudice, with complete disregard for truth or falsity?
And Johnson thinks it is true, as does Tallis. If you think it is false then what you need to do is argue against it, not cry "fallacy!" Note that you haven't managed to address Tallis' argument at all, and Tallis is defending (1).
Quoting Metaphysician Undercover
Again, you are just imputing specious motives to Johnson. I see no reason to impute such motives, and that sort of psychologism/mind reading is bad philosophy. If you have an argument, offer it. If all you are going to do is say, "I did some mind-reading and found a bad motive," then you're not doing philosophy.
I think the critique simply misunderstands Berkeley, as does Johnson's reply. The argument from the stone isn't really an informal fallacy so much as a "common misunderstanding of idealism."
People misread Berkeley and fail to see that his ontology predicts everything to be observably identical to a physicalist ontology.
IMO, the more appropriate criticism of Berkeley is that his philosophy is shallow. It's more an elaborate critique than an actual philosophy. Sure, it's a fine critique of the absolute necessity of the materialism of his day. The reigning philosophy wasn't conclusive. But in this, it shares a lot in common with arguments from underdetermination. Merely showing that your theory covers as much if the evidence as alternatives doesn't do all that much to move the needle; good philosophy explains the world. Berkeley's idealism cannot be ruled out, but it also doesn't do much to rule itself in. It seems very ad hoc.
Now granted, the critique of subsistent "matter" taken alone is stronger, but I feel like there are a lot of people who do this better.
Lots of philosophy involves God. The problem with Berkeley's invocation of God is that it seems much like Anaxagoras' invocation of Nous, an ad hoc way to plug holes.
Agree. That's why I described him as a naive idealist, although a bit tongue-in-cheek. But his commitment to nominalism and rejection of universals undermines many other aspects of his philosophy. (I read somewhere that C S Peirce wrote a review of Berkeley which agreed with him in some respects but criticized his nominalism.)
Quoting Count Timothy von Icarus
Specifically, those who came along later!
To me, thinking that such a premise is true, just demonstrates a lack of understanding of Berkeley, as Wayfarer has already (very competently, I might add) argued. So there is no need for me to address this as well.
And it still is begging the question, because no information is given as to why the premise is believed to be true. Your assertion reduces it to the following argument: "If this premise is true, Berkeley is refuted. I believe this premise is true therefore Berkeley is refuted." As you can see, the primary premise " if this premise is true, Berkeley is refuted", is still an instance of begging the question. All you are doing is trying to prove that one argument is not a matter of begging the question, attempting to justify that claim with another argument which is begging the question. Your claim is supposedly justified by an argument which is fallacious, as begging the question. Therefore, if you keep progressing in this way you'll have an infinite regress of arguments which all beg the question, because the claim that each one does not, is supported by an invalid argument (an instance of begging the question).
Clearly the charge of "begging the question" cannot be avoided in this way. If whenever someone had that charge against them, they could simply be vindicated by saying "but I actually believe that premise is true, rather than simply manufactured to necessitate the conclusion", the fallacy of "begging the question would be impotent.
Quoting Leontiskos
I believe it's very obvious that Johnson had "specious motives", and you are being ridiculous to assert otherwise. When someone says "I refute you thus", and makes an action which is intended to demonstrate refutation, it's very obvious that the person has the specious motive of creating the illusion that the argument could be refuted by an action, without actually addressing the argument itself.
Here's an example to consider. Suppose I produce an argument which concludes that you do not have the ability to climb a specific tree. You say "I refute you thus", and you climb that tree. That act does not refute my argument. It provides evidence that there is probably a problem with my argument, somehow, or somewhere, and that my argument ought to be refutable, but actual refutation requires demonstrating that problem.
This is also the issue with Zeno's paradoxes. We see that the arrow moves, and this is very strong evidence that Zeno's argument ought to be refutable. But the movement of the arrow does not actually refute Zeno's argument, which proves that movement is logically impossible.
#34 b .To all which, and whatever else of the same sort may be objected, I answer, that by the principles premised we are not deprived of any one thing in nature. Whatever we see, feel, hear, or anywise conceive or understand remains as secure as ever, and is as real as ever.
#37 .The philosophic, not the vulgar, substance, taken away.--I will be urged that this much at least is true, to wit, that we take away all corporeal substances. To this my answer is, that if the word substance be taken in the vulgar sense--for a combination of sensible qualities, such as extension, solidity, weight, and the like--this we cannot be accused of taking away: but if it be taken in a philosophic sense--for the support of accidents or qualities without the mind--then indeed I acknowledge that we take it away, if one may be said to take away that which never had any existence, not even in the imagination.
#33 . The ideas imprinted on the senses by the Author of Nature are called real things; and those excited in the imagination being less regular, vivid, and constant, are more properly termed ideas, or images of things, which they copy and represent. But then our sensations, be they never so vivid and distinct, are nevertheless ideas, that is, they exist in the mind, or are perceived by it, as truly as the ideas of its own framing. The ideas of sense are allowed to have more reality in them, that is, to be more (1) strong, (2) orderly, and (3) coherent than the creatures of the mind; but this is no argument that they exist without the mind. They are also (4) less dependent on the spirit, or thinking substance, which perceives them, in that they are excited by the will of another and more powerful spirit; yet still they are IDEAS, and certainly no IDEA, whether faint or strong, can exist otherwise than in a mind perceiving it .
#9 ..The philosophical notion of matter involves a contradiction.--Some there are who make a distinction betwixt primary and secondary qualities. By the former they mean extension, figure, motion, rest, solidity or impenetrability, and number; by the latter they denote all other sensible qualities, as colours, sounds, tastes, and so forth. The ideas we have of these they acknowledge not to be the resemblances of anything existing without the mind, or unperceived, but they will have our ideas of the primary qualities to be patterns or images of things which exist without the mind, in an unthinking substance which they call matter. By matter, therefore, we are to understand an inert, senseless substance, in which extension, figure, and motion do actually subsist. But it is evident from what we have already shown, that extension, figure, and motion are only ideas existing in the mind, and that an idea can be like nothing but another idea, and that consequently neither they nor their archetypes can exist in an unperceiving substance. Hence, it is plain that the very notion of what is called matter or corporeal substance, involves a contradiction in it .
(A Treatise Concerning the Principles of Human Knowledge, Sec 1, Of the Principles ., 1710, in
https://www.gutenberg.org/files/4723/4723-h/4723-h.htm)
-
First cherry-picked, I know. I picked out what I thought most related to the OPs query. Theres a veritable plethora of mitigating textual relevance the cherry-picking avoids, hopefully not so much as to show I missed the point completely.
Johnson, following fellow British empiricist Locke, used the primary qualities of things in order to refute the validity of mere ideas as resemblances of them, re: ideas cannot fracture a toe. But in doing that, insofar as, e.g., solidity in things is necessary for fracturing toes, he did nothing to prove such primary qualities were existents in things, the absence from which it follows, that such primary qualities remain mere ideas in the mind of the mediating perceiver, in accordance with Berkeleys considered metaphysical thesis, in opposition to Locke.
If you cant prove primary perceptible qualities in us are not ideas in an immediate principal perceiver, perhaps it can be argued .what difference would it make to the human perceiving mind, if they were not? Was the idea of measurable distance implanted in my head as an idea belonging to some sort of prevalent, re: un-constructed, spirit, or does the idea belong to me alone, as a mere distinction in relative spaces?
Sapere aude anyone?
But if any "idealist" merits some teasing (if this is warranted at all) it would be Arthur Collier. He didn't even have God as a guarantee of stability or existence of external objects.
The biggest issue here is that, for whatever reason, we have some trouble (at least I do) in understanding how concretely existing things could be solely ideas.
Seems to me to be the one thing you can't think away from objects in some form or another.
Quoting Leontiskos
When the topic of this discussion was current affairs, religion ruled the day, hence, one couldnt stray too far from it and maintain his cultural standing. It was a race of sorts, among interested parties anyway, not so much the common man, to offer the strongest arguments for the distribution of properties, whether they belonged to things given by Nature, or belonged to things given by deities. But then .Nature itself may have been given by deities, resulting in nothing new. Comparative philosophical doldrums.
Enter science proper, and stuff gets real interesting.
The interesting part? Science let it be known humans could have things, could do things, entirely on their own, or at least enough on their own to call into question isolated external causality of the Berkeley-ian un-constructed spirit type.
-
Kant called Berkeleys idealism dogmatic, meaning it was formed as a doctrine without sufficient critical examination of the warrant for doing so, and the greatest of that was the principle esse est percipi, wherein the insufficient warrant falls on what it is to perceive, as formalized in s OP, re: For Berkeley, perception encompasses the whole experience of the world as presented to the mind. Which is these days pretty much established as not the case.
The way this relates to Kant, is that, generally, as you asked, in transcendental idealism, existence is granted outright, immediately removing it from necessary reference to ideas and the condition of our perceptions. Very generally, to be sure.
Did Berkeley in the 18th century have any empirical evidence upon which to base his foresight of "modern subatomic physics" view of Matter? Or was his Idealism a> just intuition or b> expansion on Plato's metaphysics?
We now know that the table before us, that seems to be solid wood, is mostly empty space*1. So the solidity of the "substance" is a sort of illusion conjured by the mind ; but a "stubbornly persistent illusion"*2 that all humans share. That Johnson's rock will resist the impact of a foot is due to immaterial force fields*3, not to Matter in the Democritean sense. Could those invisible-yet-powerful forces & energy & gravity be interpreted in terms of the Mind of God (Universal Mind) binding the world together, perhaps by perceiving or conceiving*4 the cosmos as an integrated whole?
I don't mean to put you on the spot. I'm just riffing on a theme, and going beyond my scope of meager philosophical knowledge. :smile:
*1. Atoms are not the ultimate particle: they are nearly all empty space. This space is filled with electric and magnetic force fields. These fields are incredibly powerful, and hold electrons in their atomic prisons. The fields govern potential energy, and are strong enough to mean that atoms resist like a solid medium.
https://academic.oup.com/book/985/chapter-abstract/137840897?redirectedFrom=fulltext
*2. Albert Einstein wrote: The distinction between the past, present and future is only a stubbornly persistent illusion.
https://www.spudart.org/blog/einstein-time-stubbornly-persistent-illusion/
*3. A force itself isn't "made of" anything tangible, but rather is a concept describing the interaction between objects, resulting from the exchange of particles called bosons, and is mathematically defined as mass multiplied by acceleration (F = ma).
___Google A.I. Overview
*4. According to George Berkeley's philosophical theory, God is the ultimate perceiver, meaning that the world only exists because God is constantly perceiving it; essentially, "to be is to be perceived," and since God always perceives everything, even when humans aren't, the world continues to exist even when no human is observing it
___Google A.I. Overview
Yes, I think this is the key problem for most people in thinking this matter through.
The idealist might say that the idea of solid objects misses the point and remains stuck in a framework of metaphysical realism. In Johnson's case, the toe and the breaking themselves are a product of consciousness. I assume that the point of Berkeley is that the world of primary qualities does not exist independent of the mind. Solidity and the notion of 'hard matter' does not exist independently of mind and so kicking the rock, breaking a toe are mental experiences. It is how consciousness appears when experienced from our perspective. The solid stone and the foot's impact upon it are examples of the ability of consciousness to create a coherent world of experience - held together in the mind of God. Or in the case of Kastrup - we are all participants or aspects of a 'great mind' which is the source of all reality.
Of course. But I'm of the view that it was this emerging modern view of the universe that the good Bishop wished to oppose. That the reason idealism as school of thought begins to appear in this time, is because of the rejection of scholastic realism, which held that particulars did not posses their own inherent or intrinsic reality, as scholastic realism held that the being of particulars was grounded in their intelligible form. Whereas the emerging forms of nominalism held that particulars are real 'in their own right', so to speak. This has had many consequences, most of which we're not aware of, as they are formative in modern culture.
Quoting Gnomon
It's important to get what Berkeley is saying. Many people, even many philosophers, take him to be saying that solid objects are all 'in the mind', which is why Samuel Johnson believed that kicking a rock refuted his arguments. As I've been saying, that is based on a misunderstanding of Berkeley's contention, which was that there is no material substance apart from all of the perceived attributes of objects (size, shape, weight, solidity among them.) So he's not saying the rock doesn't exist, or is a 'mere' idea, but that what we know of it, is the sensible impressions it causes in us. As per the paragraph above on the meaning of 'substance' in Berkeley - the meaning of substance is crucial in this context. It doesn't mean 'a material with uniform properties' (a sticky substance, a waxy substance, a very hard substance.) It means something like 'a particular of of which attributes can be predicated, or in which attributes inhere'.
(The point I'm interested in, is that 'substance' was derived from the Latin translations of Aristotle's 'ousia' in his Metaphysics, and that is a form of the verb 'to be'. So it is at least arguable that what philosophers often refer to a substances, might be better rendered as 'beings' or 'subjects'. It's not entirely correct, but it conveys something important. For instance, in translations of Spinoza, we read 'God is the infinite, necessarily existing, unique substance.' What if that was given as 'subject' or 'being'? Again, not quite right, but conveying something that has been lost in translation, and which leads to the idea, mistaken in my view, that 'substance' is objectively existent as a kind of thing, no matter how ethereal. But at any rate, it is the philosophical notion of 'substance' and in particular 'corporeal substance' which is at issue.
Of course it is true that Berkeley had no conception of modern physics, although he might well have known of ancient atomism. But it is arguable that modern physics has also undermined the conception of 'corporeal substances'. It has certainly cast doubt on the conception of the mind-independence of fundamental particles, at issue in the 'Bohr-Einstein debates'.)
Berkeley held to Platonism in some ways, with the emphasis on ideas, but contra in others, as he opposed universals. Many say that is the real shortcoming of his philosophy.
The textbook account of Kant on Berkeley is that, after the first edition of Critique of Pure Reason, Kant was angry that many critics took him to be affirming Berkeley's basic thesis. Accordingly in the B edition, he included a section on the Refutation of Idealism, directed at Descartes and Berkeley. You can find an account here.
Like 'substance', I think 'idea' in philosophy means something other than the parade of thoughts, words and images that pass the mind's eye. Objects are recognised by us as kinds and types - this is where Kant comes in - and without that recognition, which is part of the process of apperception, then they would be nothing to us. Experience presents itself to us in the form of ideas. It is much more clearly enunciated by Schopenhauer, in the opening paragraph of WWI, where he recognised Berkeley's 'permanent service to philosophy', although then immediately saying 'even though the rest of his teaching should not endure'. (Talk about a back-handed compliment.)
Quoting Mww
Yes, but is it just modern science? Because there is plenty of philosophy between Plato and Berkeley that manages to avoid Berkeley's idealism. I'm sure if I investigated the way that Berkeley was reacting to Locke I would have a better understanding of this issue.
---
- Okay, thanks. :up:
I think science did more than anything else to liberate the intellect, yes.
-
Quoting Wayfarer
Im not sure he could do otherwise, could he? I guess Im of the mind that, rather than oppose science, his raison detre was to uphold religion. I mean .
..But you will say, has Nature no share in the production of natural things, and must they be all ascribed to the immediate and sole operation of God? I answer, if by Nature is meant only the visible series of effects or sensations imprinted on our minds, according to certain fixed and general laws, then it is plain that Nature, taken in this sense, cannot produce anything at all. But, if by Nature is meant some being distinct from God, as well as from the laws of nature, and things perceived by sense, I must confess that word is to me an empty sound without any intelligible meaning annexed to it. Nature, in this acceptation, is a vain chimera, introduced by those heathens who had not just notions of the omnipresence and infinite perfection of God .
(Ibid 157)
..YIKES!!!! Nonetheless odd as hell, I must say, that had I lived in 1710, I might have just as similar an opinion, as the different one I do have.
I can't help be reminded of:
Quoting TLP 6.371
I mean, for hundreds of years Christian theologians had been incorporating Aristotle into their work and refusing Occasionalism, which is pretty close to what Berkeley promoted. See, for example, Aquinas on secondary causality. To say that Berkeley is not representative of Christian theology up to that point would be an understatement. And isn't Berkeley reacting primarily to John Locke, who was himself religious? Berkeley may have been opposed to realism, but that doesn't mean religion is opposed to realism.
Very interesting responses. So am I right in thinking that for you idealism consists more of our cognitive apparatus making order our of a type of chaos (but there is some sort of "noumena" to begin with)? I don't read you as subscribing to the notion that there is only pure consciousness and nothing else, held by a ground-of-being style great mind, in which we are all participants or instantiations.
Sure, no prob. The human can only account for his world in his own terms, and whatever the difference between his terms and Natures, cannot be determined by them. The illusion resides in thinking they can.
Right? Is this somewhat like what reminds you?
I think I'll defer again to Schopenhauer. While he came along much later than Berkeley, his insight no object without a subject encapsulates a key idealist critique of materialism. Schopenhauers analysis deepens Berkeleys argument by revealing how materialism presupposes the subjects forms of knowledgetime, space, and causalitywithout acknowledging their dependence on the subject. This convergence between their critiques underscores the enduring relevance of idealism in challenging naive realism and materialism, which is, as Schopenhauer often insists, 'the philosophy of the subject who forgets herself'.
[hide="Reveal"][/hide]
Yes. And that is an argument that can be given. I don't disagree with some varieties of idealism by the way, like Kant's, or Burthogge's or Cudworth's.
But that aside, I can think away everything I can about an object, including my concepts, my sensations, everything I can think of. I still have the intuition that even if I don't feel it, I cannot pass through walls, something is there that is not solely mental.
But as you say, this can be explained by being a content of consciousness. It's nebulous.
Quoting Leontiskos
A fun quote from Peter van Inwagen:
Lots commendable in the WWR excerpt, but for this:
Each conscious being indeed maintains the form, the condition, of its world in accordance with its effects, but each conscious being isnt his own worlds existential causality.
That being said, theres agreement whereby *reductive* materialism, as a purely monistic ontology, ignores the subject in favor of the regressive series of things.
Im not sure realism has much to do with it, whereas the primary source of it, its fundamental causality, does.
. Such is the artificial contrivance of this mighty machine of nature that, whilst its motions and various phenomena strike on our senses, the hand which actuates the whole is itself unperceivable to men of flesh and blood .
(Principles . , 1710, #151)
To understand this "what difference would it make...", we need to get a good grasp of the philosophical concept itself, "matter"; that being the concept which Berkeley insisted we can dispense with.
In my understanding "matter" is a concept employed by Aristotle to underpin the observed temporal continuity of bodies, allowing for a body to have an identity.
Hegel, in positing the priority of "the Idea", rejects "identity" and this enables his logical dialectics, characterized by "becoming". That takes the world of change, and brings it from the external material realm, into the internal Ideal realm. Instead of being eternally unchanging, the Idea evolves with time. This is a significant change, but whether or not it provides the means to completely dismiss the concept of "matter" is debatable. Marx, who followed Hegel, argued that under Hegelian principles "matter" is still necessary as the kernel of content within the Idea.
That's why begging the question is known as an "informal fallacy".
Agreed.
Quoting Metaphysician Undercover
Kinda agreed. Id be more inclined to grant to the concept of matter the underpinning for spatial continuity allowing a body to have an identity.
Quoting Metaphysician Undercover
I understand the concept represented as becoming, and, with respect to the kernel of content within the Idea, isnt that more Platonic? Maybe where the notion of becoming initiated? My armchair mandates that matter is the kernel of content for experience; ideas, as such, have no material content at all.
What is "science proper?"
To me, it's always seemed a bitter irony that just as there is an explosion in scientific progress (helping to drive the "Great Divergence" of the 19th century whereby Europe became much wealthier and militarily stronger than Asia), much philosophy of science seems to become incredibly dismal, consumed with skepticism. Even today, "philosophy of physics," "philosophy of biology," "philosophy of complexity," or "philosophy of economics," are filled with interesting ideas, whereas "philosophy of science" often takes the form of dull reductions of science to "observation + modeling," scientific understanding to "prediction," and knowledge of causes to "more prediction."
While it's true that for Aristotle "matter is what stays the same," when there is change, the "matter" and "substance" of Berkeley's era had changed dramatically from their ancient or medieval usages. The entire idea of "materialism" makes no sense from an Aristotelian framework. It would amount to claiming the whole world is just potency, with no actuality, and so nothing at all. But the term "matter" by Berkeley's era is more often conceived as a sort of subsistent substrate (often atomic) of which spatial, corporeal bodies are composed, such that their properties are a function of their matter (which would make no sense under the older conception of matter as potential). By way of contrast, Aristotelians would speak of the "material intellect" of the soul, the matter of abstract mathematical objects, the form of a logical argument (from whence we get "formal logic") versus its matter, the phrase "subject matter," etc.
"Idealism" ("eidos-ism") would also make no sense in the Aristotelian frame. Saying "everything is idea" would be to declare that everything is act, which would preclude change, essentially putting you back with Parmenides. This is why "idealism versus materialism" is a modern debate (although it has some loosely analogous precursors). Aristotle might be close to Berkeley in some sense, in that the world is intellect, but this is taken in a very different (and IMO far more developed) way. The redefinition of substance also looms large here, since materialism versus idealism can be framed as "everything is material versus mental substance," a distinction which required the radically different early modern notion of substance to make much sense.
Locke's matter, for instance, is closer to ancient elements than Aristotle's matter.
.dull reductions to "observation + modeling ..sounds about right to me. Id add in experimenting, and the whole process doesnt have to be dull, necessarily. Although dunno if I could sit still long enough waiting for a cosmic neutrino.
Isn't it interesting, though, that with David Hume and the advent of modern philosophy, the whole concept of natural causation is thrown into question. I recall Bertrand Russell saying in the History of Western Philosophy that Hume's scepticism would even cast science itself into doubt, had not Kant 'slipped a plank' under it. He was referring to Kant's 'answer to Hume', whereby Kant resurrected causal relations by showing that they are among the necessary conditions of reason.
I recall an exchange some years ago between Richard Dawkins and the now deceased Bishop George Pell about evolution and creation, the transcript of which was kept online by the Australian Broadcasting Commission:
Dawkins unwittingly expresses the way in which his type of scientific materialism is irrational, as it has no concept of causation beyond the material. There is no reason for anything existing, only a kind of forensic reconstruction of prior temporal events.
In that passage Schopenhauer makes explicit what is implicit in Kant: causation is not an inherent property of the objective domain, but a necessary condition of how the mind structures experience. The logical relations and causal connections we discern in the world are only possible because the world is ideaa representation shaped by the mind. For the empiricists, this connection between causation and logic is severed, leaving causation as little more than a psychological habit without grounding. Schopenhauer restores this connection by showing that causation exists because the world exists as idea.
Modern thought generally assumes that temporal priority subsumes causal or explanatory priority, but this is far from self-evident. As Schopenhauer argues, time and causality are structures in consciousness, not independent realities. To conflate what comes first in time with what is most fundamental in being is to mistake the descriptive for the ontological. For instance, while the Big Bang may precede the universe temporally, it does not answer the ontological question of why there is something rather than nothing. Similarly, while neural activity may precede consciousness in time, it does not explain the existence of consciousness, which is the very framework through which we understand causation.
This is exactly what Schopenhauer is saying: 'The objective world, the world as idea, is not the only side of the world, but merely its outward side; and it has an entirely different sidethe side of its inmost natureits kernelthe thing-in-itself But the world as idea only appears with the opening of the first eye.'
That comment is true, but the "something" is not necessarily Matter, and may even be a form of Mind. If the notion of mental matter sounds odd or woo-woo, it's understandable. So, I'll try to explain, but Science and Philosophy tend to focus on opposite sides of this equation. Therefore, this post is a cross-over.
According to modern physics, the something blocking your attempt to pass through the wall is not Matter per se, but Atomic Forces interpenetrating the space between sub-atomic particles : electrons, protons, etc. Those binding & repelling forces are what gives the "appearance of solidity to pure wind"*1. It's the sensation of push-back that makes the wall seem solid, even though its atoms are now known to be 99% empty space. Therefore, Johnson's rock and his shoe were mostly matterless, yet those invisible binding forces cause his foot to bounce-back without penetrating the apparent surface of the stone.
But, what is a Force or Energy? It's not a material substance, but a positive (push) or negative (pull) relationship (statistical ratio), and exists in Potential (available) or Actual (causal) forms*2. And the knowledge of conceptual relationships (yes/no ; on/off ; hot/cold) is what we mentally interpret as meaningful Information*3. Potential (theoretical ; imaginary) energy has no sensable form, but Actual energy can even take on the form of Matter : E=MC^2. Yet we sense Matter directly only as the sensation of weight due to mathematical Mass, or indirectly by the stimulus of reflected light from the force field around the atoms, or by repulsion of a foot, when it attempts to pass between a material rock and an apparent hard place.
What I'm trying to say here is that the "appearance of solidity", and the sensation of weight, and the visual image of a rock, are all mental functions. If you see a gray mass, and you believe it to be solid & massive, you will refrain from kicking it. Unless, of course, you are trying to demonstrate that something is there "that is not solely mental". You know from personal experience that your mind/body requires a door in order to "pass through a wall". And yet, Quantum Physics has revealed that the subjective Mind can be a causal force*3*4 in sub-atomic physics.
A New Age interpretation of quantum subjectivity --- as illustrated in the movie The Men Who Stare at Goats {video below} --- concluded that since the wall is nothing-but emptiness, it's only an obstacle to those who believe in matter. Conveniently ignoring the real world role of forces. Mass is indeed an abstract mathematical concept, and Matter is a lump of information relationships, but Physics is more-than just an illusion : it keeps us from falling through the floor.
The bottom line here is that Energy/Force/Causation may be a primitive relative of what we understand as Mind*5. Hence, Mind & Matter may both be forms of essential Energy. This Energy-Mind relationship is not well known*6. But, as the 5b link says : "This theory has implications for transforming states of mind and the ethical treatment of all living beings". I apologize for getting so technical, but the relationship between Matter & Mind is a fraught question on this forum. So it might help to get down to fundamentals. Or not . . . :nerd:
*1. Politics . . . according to Orwell, "is designed to make lies sound truthful and murder respectable, and to give an appearance of solidity to pure wind"
https://en.wikipedia.org/wiki/Politics_and_the_English_Language
*2a. A force is simply the transfer of energy between kinetic and potential. Energy can exist in so many forms, but the only way between those two is with a force.
https://www.reddit.com/r/explainlikeimfive/comments/wyrmvk/eli5_what_is_the_difference_between_a_form_of/
*2b. Force is what accelerates a mass. Energy is a completely different thing, the potential to create a force across some distance.
https://www.reddit.com/r/explainlikeimfive/comments/wyrmvk/eli5_what_is_the_difference_between_a_form_of/
*3. Energy is the relationship between information regimes. That is, energy is manifested, at any level, between structures, processes and systems of information in all of its forms, and all entities in this universe is composed of information. . . . . Information is a statistical concept, also in telecommunication engineering, say. It captures the scientific aspect of information, though not its subjective value for human beings.
https://physics.stackexchange.com/questions/22084/how-is-information-related-to-energy-in-physics
*4. Quantum subjective causality is a philosophical and theoretical framework that explores the role of causality in quantum physics. It combines ideas from quantum information, computer science, and general relativity to explain how causality and time work in the quantum realm.
___Google A.I. Overview
*5. The statement "mind is energy" means that our thoughts, feelings, and consciousness can be understood as a form of energy, as the brain's activity generates electrical impulses and chemical reactions which are essentially energy in action, allowing us to think and experience the world around us; essentially, our mental processes are not separate from physical energy within the body.
___Google A.I. Overview
*5b. The mind is viewed as energies of relationships, with no beginning and no end, that give rise to consciousness in an observer processing change or information from the universe.
https://researchoutreach.org/articles/mind-as-energy/
*6. Interactionism :
In his 1996 book The Conscious Mind, David Chalmers questioned interactionism. In 2002 he listed it along with epiphenomenalism and what he calls "Type-F Monism" as a position worth examining. Rather than invoking two distinct substances, he defines interactionism as the view that "microphysics is not causally closed, and that phenomenal properties play a causal role in affecting the physical world." (See property dualism.) He argues the most plausible place for consciousness to impact physics is the collapse of the wave function in quantum mechanics.
https://en.wikipedia.org/wiki/Interactionism_(philosophy_of_mind)
*6b. Type-F monism is the view that there are phenomenal or at least protophenomenal properties that underlie physical properties like mass and charge. This is a version of panpsychism.
https://philarchive.org/archive/BRODCO
Eddington's Two Tables
This is an interesting perspective. But I tend to apprehend spatial features of a body as formal rather than material. Spatial features tend to be the traditional properties, which are formal.
In his Physics, Aristotle describes the material cause as what persists through change. This idea of persistence from one time to another, is why I interpret "matter" as temporal continuity. Zeno's paradoxes, and the idea of infinite divisibility, had cast doubt toward the reality of spatial continuity. So Aristotle moved to assign identity to the thing itself, and the essence of being a thing is to have temporal extension, as a continuous duration of being.
Quoting Count Timothy von Icarus
I do not agree. Berkeley takes "matter" in very much the way of Aristotle. That's how he manages to conceive of substance without matter.
Quoting Count Timothy von Icarus
I believe that Berkeley is actually demonstrating the incorrectness of this 'new' way of conceiving of "matter" by showing how these ideas that people have about "matter" do not hold up if we adhere to principles.
The reason why I mentioned Marx, is because he also maintained the Aristotelian conception of matter. But Marx demonstrated how, contrary to what you say here, a true materialism is possible. It just develops some odd features like violation of the law of non-contradiction in dialectical materialism. But Marxist materialism is actually well structured conceptually.
Quoting Count Timothy von Icarus
This is why Marx was so easily able to invert Hegelian idealism, and transform it into materialism. Aristotle refuted traditional (Pythagorean) idealism by putting human ideas, and matter, both in the same category as potential, then applying the cosmological argument. So when Hegel proposed "the Idea" as fundamental, Marx was able to apply Aristotelian principles to replace "the Idea" with "matter", thereby hijacking Hegelian dialectics and turning that idealism into materialism. And then it turns out that Hegelian principles are better suited to materialism than idealism.
I agree with @Count Timothy von Icarus. As I put it in an earlier post:
Quoting Wayfarer
Note the 'and transformed'. Berkeley was very much at odds with Aristotelian universals, which he rejected as 'abstract ideas'.
That's fine and you are right that the matter we have now is just extraordinarily removed from our common conceptions of it. This is true and sure; I do agree the world is a construction of the mind. We don't even need metaphysics to establish this, I think it just follows form facts of the matter.
The issue is that our everyday image of the world is not and probably cannot, be exhausted by whatever physics says about it. From an "everyday" perspective, you can "think away" all sensations and maybe most conceptions, but I, that is me specifically, have trouble removing solidity from this, it seems to me that based on manifest world terms, we still would bump up against something, even if we can't feel it.
This is different that speaking about the micro-constituents of matter, as I see it,
The quote you provided seems to agree with me. Berkeley was criticizing the 'new' conception of matter. And he did this by falling back onto a more Aristotelian conception which allows him to disassociate substance from matter. He showed that substance does not require matter. This is what i said:
Quoting Metaphysician Undercover
On the upside, science itself has seemed remarkably resilient in the face of this drum beat. I worked at a place that specialized in glaucoma for a while and the doctors would make pitches about "fully understanding the causes of this disease." Likewise, physicists have not been deterred from subtitling their books things like: "the quest for the ultimate nature of reality," or "what is real?" etc.
Yes, I'll concede that, but there's nothing in Berkeley's philosophy that corresponds with the 'morphe' of Aristotle's hylomorphism. But you're correct in saying that he is targetting the conception of matter held by the other early modern philosophers.
I have a number of such books, although they're by science writers rather than physicists (Manjit Kumar, David Lindley, Adam Becker.) But the fact that it's still a question is rather in Berkeley's favour, don't you think?
Quoting Wayfarer
Id chalk that natural causation question up to QM rather than philosophy. Whether the cause/effect principle is resident in the human intellect, such that natural causation is comprehensible, that I would attribute to a changing philosophical agenda.
Quoting Wayfarer
Absolutely. Hence the Kantian dictate that both are equally necessary conditions for empirical knowledge. The antinomies prove one can be nonetheless thought as ulterior or antecedent to the other.
Quoting Wayfarer
The logical relations and causal connections we discern are a product of our intellectual capacity and only possible therefrom, having to do only with existent objects in relation to each other, or to ourselves. When the world as idea is thought as a universal concept, it is not necessary for us to discern the logical relations of its particular content.
-
Quoting Wayfarer
It bothers me that world as idea only appears. The world may indeed be an idea, a universal object of reason, re: that in which are found all possible objects but is not itself an object, but world as idea isnt that which appears to an eye. Only existences appear, the world, as pure transcendental object, isnt an existence.
Im sorry, but opening of the first eye is absurd, if such is meant even remotely literal. To reconcile the absurdity, we are forced to admit the metaphor merely represents some arbitrary initial impact on a fully developed rational intelligence. The problem for humans then reduces to the opening of the first eye may not have even been human, but the world as idea is predicated on it anyway, which is a contradiction. Nothings solved by attributing first eye to humanity in general, nor to individual human subjects therein.
-
How is the ding as sich not just as outside as the objective world? The inner kernel of an outside thing is just as outside as that of which it is internal.
Hmmm .dammit, youre right, I forgot about that. In the strictest possible sense of spatial continuity, yours is the stronger for being deferred to the temporal, but for the common understanding of the ordinary man of which there are decidedly many more than philosophers per se .that a thing is in his way is very much more apparent than the notion that if he waits long enough, it wont be.
I didn't say that he lays out a new theory. I said that by adhering to the Aristotelian (traditional) principles, for understanding "matter", he shows how the divergent conception of matter, what you called "subsistent substrate", is misdirected. "Matter" in this sense is conceived as necessary, while the traditional conception classed it as potential. Hume, I believe had a similar approach. "Matter" in his philosophy would be an inductive concept, therefore lacking in necessity.
Maybe Wayfarer is more accurate, and Berkeley was showing problems with that 'new' concept of matter without any understanding of the Aristotelian concept, but he seems to have at least a fundamental grasp on the classical understanding of contingency which is Aristotelian based.
I believe the shift away from Aristotelianism, in the way that "matter" is conceived, is derived from the physicists. They were the first to drop Aristotle as a principal source for higher education, while his logic was still taught in schools, and philosophers would still study Aristotle.
Newton assigned to bodies, the fundamental property of inertia, with his first law of motion. This effectively replaced Aristotelian "matter", as what provides for the substance of a body. The logical consequence of this, which is unseen without critical analysis, is that "inertia" is a property, while "matter" in the traditional sense is in a separate category, properties being formal. So "inertia" cannot really replace "matter". Therefore it was assumed to be implicit within that conception of inertia, that inertia is a property of matter. But that leaves matter as something itself, subsistent. I believe that this is how the divergent concept of matter, as "subsistent substrate".
A similar, but more substantial issue has developed with the concept of "energy". Strictly speaking, "energy" refers to a property. However, when it comes to things like electromagnetic energy, radiation, people often speak of "energy" as if it is a thing itself. This leaves energy as a property without a substance. We cannot assign "matter" as the subsistent substrate which energy is a property of, so that logically, this concept is left unsubstantiated. This is the result of moving from Aristotelian "substance" which is the fundamental property of individuals, particulars, to allowing that "movement" is what is fundamental to particulars. Movement is allowed to replace substance, but in doing so the need for something which is moving is lost.
Quoting Wayfarer
I think, that at Berkeley's time, modern philosophy was extremely underdeveloped. There was Cartesian "mind", and Hume was discussing "ideas", but very little in the way of extensive understanding of metaphysics and ontology as there was with the Scholastics. The reason I believe is that popular focus had turned to physics, and a rapidly evolving understanding of motion. Attention was turned in this direction, so the primary focus of philosophy was actually the mathematics required to support the new physics.
Berkeley, I believe noticed that physicists were doing strange things with the conception of "matter", and he wanted to bring attention to this. However, he does not appear to have had any extensive training in classical philosophy.
Quoting Mww
It's more than just the ordinary man who believes in the priority of spatial relations over temporal. Modern physics has reduced time to being an aspect (dimension) of space. This is due to the fact that with empirical science we assign importance to sense observation in understanding the reality of substance. Sense observation is of the external, therefore producing principles of spatial separations and movements. "Time" as being understood through internal reflection, and logical comparisons, is secondary.
So what happened to spatial movements making the concept of time necessary, rather than merely secondary?
Where can I read about the reducing of time to an aspect of space?
Yes. There's only one table, but there are two different ways of looking at, or thinking about, the table. One perspective is scientific (particles & forces in space) and the other philosophical (appearances & phenomena). Scientists use artificial extensions of human senses in order to study the hidden world beyond surface appearances. Philosophers use the scientific information to look inside the human mind, and to imagine how meta-physical ideas relate to physical reality. :smile:
PS___Since I often get negative feedback for my unconventional use of the term Metaphysics, here's a more modern definition :
Mental Meta-Physics : Beyond the physical :
Metaphysics, by definition, deals with concepts that go beyond the physical world, so "metaphysical mental" implies examining the mind in a way that isn't solely limited to its neurological functions
___Google A.I. Overview
Yes. Biology & Physics give us a look inside the skull of an observer. From those facts we can construct a mechanical model of how the brain produces ideas. However, there remains an unexplained gap, between neuronal networks and mental functions, that Meta-physics can bridge with reasoning & imagination*1. :smile:
Note --- See the modern definition of Metaphysics and Wayfarer's link Eddington's Two Tables in my post above.
*1. In philosophy, reasoning with imagination is a type of reasoning that uses imagination to draw conclusions from existing evidence. It's a distinct way of reasoning that's not the same as reasoning with doxastic {belief} states
___Google A.I. Overview
Logic can't produce evidence but it can rationalize it. Immaterialism is a practice of the mind's ability to rationalize. For Berkeley, this was more of a way to do mental push ups to strengthen his ability with empirisism. Berkeley doesn't consider the external world any less real. The people who take it that far and get frustrated by it fall in to his little mental game/exercise and I could rightly see him chuckling to himself in a fit similar to a mischievous child pranking the minds who perceived his works on immaterialism too literally. Berkeley's immaterialism was an exercise in overcoming of oneself in their opposite in my opinion. Much like Nietzsche and Zarathustra...
.
Isn't this where the colloquial "go kick rocks" comes from? We say it to those who don't "get the picture" in otherwords. Absolutely hilarious to me picturing Berkeley in his jamjams muttering and cackling to himself "go kick rocks..." as he's writing down some ideas he knows will put a twist in someone's britches...
Though, interesting to me is that it seems Berkeley focused heavily on the opposite ends of the Apollonian spectrum. Where as Nietzsche's balance was between the Apollonian and Dionysian.
It is common knowledge that in the cosmic scheme, h.sapiens has only existed for the merest sliver of time, and mammals and higher animals generally relatively recent arrivals. That is a matter of temporal sequence, but again the observing mind provides the framework within which that is intelligible. And as I've brought Schopenhauer in, I'll double down:
So, while it is an empirical fact that universe pre-existed conscious beings, the way in which it exists outside of, or before, conscious beings is unknowable as a matter of principle, as the knowledge we have of it, which is considerable, is still held within that intellectual framework. That is why the great Kant could say that one could be both an empirical realist AND transcendental idealist and see no contradiction between them.
Quoting DifferentiatingEgg
No, it comes from the Samuel Johnson anecdote, which is described in the OP.
Quoting Metaphysician Undercover
Plainly, the death knell for Aristotelianism was the advent of Galilean science and the collapse of the 'medieval synthesis.' And Descartes and all the early moderns took great pains to differentiate themselves from 'the schoolmen', on the not unreasonable grounds that it had become stultifyingly dogmatic. (Actually I still remember an anecdote from the very first lecture in philosophy I attended, by Alan Chalmers, author of What is this Thing called Science? He related the story of group of monks who fell into an argument about how many teeth horses had. They all scurried off to the library, but alas, when they reconvened, they reported that as this fact wasn't in Aristotle, then it couldn't be known. When one fellow suggested going and actually looking in a horses mouth, he was ridiculed.)
I was referring to that event in the OP. Apologies for the poor clarity there.
This is a conflation between our ability to discern characteristics of things and the characteristics themselves.
Agreed; couldnt be otherwise. Im a little particular about descriptions of what there is to work with, those necessary conditions, and how they are treated.
What does this imply about the body then?
Again I'm impressed with Schopenhauer's attitude (although recognizing the thread is about Berkeley).
Schopenhauer sees the body as the one phenomenon we know from both the first-person (inside) and third-person (external) perspectives. Unlike other objects, which are only known to us as representations, our body is directly felt as Will. Accordingly, bodily actions are not caused by will in a mechanistic sense; rather, they manifest the will. When I move my arm, it is not that my will causes the movementit is the movement. In this respect the body provides an analogy for understanding the nature of a larger reality: things appear as representations, but in their essence, they are Will.
That is similar to how analytical idealist Bernardo Kastrup puts it:
Quoting Mind over Matter
I appreciate that answer as it gives quite an informative reference towards Nietzsche's "Will to Power," and what he had been considering.
Guy tried to say
"It's hard to have basic consistency with you, I'll be Kantian by the time I'm done."
I responded with "You're throwing out red herrings because you're afraid of answering the question cause you'll invariably have to use what I said originally and you tried to refute... and I had a discussion today that allows me to smh at your perception of Kant..."
And proceeded to dunk on him some more with what I mentioned on Berkeley and what you mentioned about Kant. That was a lot of fun ty!
Spatial movements are what make 'the concept' of time necessary. But don't you believe that there is something real which the concept represents? This would be what we know as "the passing of time". Aristotle distinguished two senses of "time". The primary sense is "time" as a measurement, a number assigned to a motion in measurement. This would be 'the concept' of time. But he also explained how in another sense, "time" refers to the thing measured.
So for example, "it took an hour for me to drive to work today" would indicate the primary sense, a measurement. But "an hour past while I was driving to work today" indicates the secondary. Notice the difference? In the first there is a measurement and this measurement assigns "an hour" as the duration of that act. In the second, there is a chunk of time which passed, measured, and this is called "an hour". In the first, the thing measured completely drops out of the picture as irrelevant, because all that matters is the measurement.
Quoting Mww
This is relativity theory. It's known as spacetime, in which time becomes the fourth dimension of space.
https://en.wikipedia.org/wiki/Spacetime
Ok. Thats all I was asking.
-
Quoting Metaphysician Undercover
. In order to have a complete description of the motion, we must specify how the body alters its position with time; i.e. for every point on the trajectory it must be stated at what time the body is situated there
(Einstein, Relativity .., 1. 3., 1916, in Lawson, 1920)
I dont find the justification for the given alters position with time, with your fourth dimension of space. Besides not needing to delve into non-Galilean parameters.
I've read Eddington's book, it's very good and very much readable even today.
But I'd say this is more closely associated with epistemology than metaphysics. There is always going to be a metaphysical component in epistemology, but it's quite small.
One of the two tables is certainly a part of metaphysics. They everyday table, less so.
Doesn't epistemology rely upon metaphysical commitments for it to make sense? I'm not sure one can meaningfully talk about what we can know unless we have resovled what there is and somehow we continually end up in a tail chasing discussion about whether an external world exists outside our perception and what it is. Not to mention the quesion of time and space - are they products of the cognitive apparatus of human minds, or do they exist? Don't scientists subscribe to a massive metaphysical commitment, that reality can be understood?
Is that a massive commitment? It seems to me a matter of rather routine observations.
Well that makes sense if you believe in scientific realism - that there is a reality which can be understood and studied. This assumption isn't demonstrable by science but is taken as a foundational premise that makes scientific investigation possible.
Note, I am not saying science can't provide us with pragmatic and useful interventions in the world. I would just never mistake if for absolute truth. I think of science as more instrumental or pragmatic.
The scientific method relies heavily on limiting the kinds of questions it tackles to those that can be meaningfully addressed within a defined scope. Consider for example the laws of motion and Galileo's definitions of physics in terms of the measurable attributes of bodies. Galileo made revolutionary discoveries in the understanding of motion, including the concept of inertia (that objects in motion tend to stay in motion unless acted upon by a force). Galileo emphasized observation and experimentation, laying the groundwork for the scientific method. Newton took these ideas further and formalized them into his three laws of motion, which are fundamental to classical mechanics, meticulously following Galileo's method in collecting data and formulating theories based on evidence. This is why they are considered two foundational figures in modern science. But note that this already relies on some fundamental assumptions and axioms, notably idealisation and abstraction. The practice of physics assume ideal forms, frictionless planes, dimensionless points, and bodies with precisely measurable attributes and behaviours. On the one hand, this proves incredibly powerful in control and prediction within its domain, which is universal in principle, but on the other, it is limited in practice by the fact that the real world does not actually comprise ideal forms and measurable forces, although this method also enables fantastically high levels of approximation. And of course there's no question of its power - you're literally looking at its results!
But notice that among what this excludes is - the subject! There is no conceptual space in all of this for the actual scientist. Which in some sense is what Bishop Berkeley is attempting to restore. He's saying something like, look, unless this is real for someone, then what kind of reality does it have? Phenomenology was to bring all of this out and make it explicit, but the germ of the idea is there in Berkeley (and Descartes for that matter, who is often credited as the forefather of phenomenology.)
If we then ask how to define this mind, the answer is that it cannot be defined in objective termsbecause it is not an object. Mind does not appear to us as a phenomenon, but appears as the observer. It is the first-person to whom all experience occurs, meaning that it is never something we can stand outside of and conceptualize as we do with other objects (on any scale). As the old Hindu saying has it, to do so is like the hand trying to grasp itself, or the eye trying to see itself. This is why materialists like Dennett, recognizing that mind cannot be an objective entity, attempt to eliminate it altogether rather than acknowledge its unique status, which undermines their core tenet of the supremacy of objectivity.
This brings us back to Berkeleys critique of materialism, the assumption that only observed phenomenathe measurable and quantifiableis real. Because mind itself is never an observed phenomenon, the materialist concludes that mind must be either an illusion or an emergent property of physical processes. This, however, is an assumption, not a conclusion[sup]1[/sup]. If anything, the inverse is true: the very concept of an objective, external reality depends on the presence of an observer for whom reality appears in the first place.
This position doesnt entail a theistic frameworkits simply the recognition that experience always occurs for a mind, hence the indispensability of the subject. Whether one frames this in terms of Berkeleys God, Kastrups Mind at Large, or Husserls transcendental subject, or even contemporary enactivist approaches to cognition, the underlying point remains the same: the world appears only as structured within awareness or consciousness, within which which the subject of experience is an ineliminable pole.
----
1. 'The world is not conclusion/a species stands beyond/invisible as music/but positive as sound' ~ Emily Dickinson.
They do - there is a world to which epistemology aims to establish knowledge claims of. But if we take metaphysics to mean, narrowly, the nature of the (mind-independent) world, then making claims about ordinary objects like tables or chairs are not metaphysical claims.
These pertain to the mode of our cognition.
Yes, science is metaphysics - in large part, not entirely - because it tries to tell us what that nature is.
In any case, we do not - and cannot - go beyond appearance.
Quoting Manuel
I tend to agree but I guess that depends upon what we mean by appearance - in recent history we have certainly devised instruments that allow us to go beyond (ordinary) appearance and these tools seem to tell us that solid matter is almost entirely empty. And let's not get into quantum speculations.
And in a separate vein, is it not the case that people who claim to be enlightened are able to see beyond appearances, at least in part? Is this not a goal of mediation, etc? I'm not personally in the higher consciousness business but I am curious about the framing of these things.
You put it very well, I understand the reasoning and I am sympathetic. And what smatterings of phenomenology I have read certainly resonates.
Not necessarily what it is but how it appears - and it's an important distinction.
Neils Bohr: Physics is not about how the world is, it is about what we can say about the world.
Werner Heisenberg: What we observe is not nature herself, but nature exposed to our method of questioning.
Quoting Manuel
I'd be careful there, it's a big statement!
:pray:
To be more specific, it is Einstein's principle of the "relativity of simultaneity", which allows time to be the fourth dimension of space. This provides for "spacetime" where time is the fourth dimension of space.
By conceiving "temporal position" as relative rather than absolute, the conceived flow of time is dependent on spatial references. This allows an equivalence between spatial distance and temporal distance enabling transformations. The need, or purpose, of the "relativity of simultaneity" is to establish light speed as a constant within the conceptual framework of relativity.
But physics is also appearance too, just a much more refined attempt to make sense of the data of sense. Remember physics is mathematical because it tells us about the structure of the things that make the world, but the inner nature of these things is not revealed, we probably can't reach the "bottom level".
As for enlightened people, no, at least I don't think so. It doesn't have anything to do with lack of training or practice or perceptiveness, it's related to the nature of our faculties. We can't step outside what we see to verify whatever it is we see.
Likewise, we cannot leave our experience to see what may be behind it.
Quoting Wayfarer
I agree.
But we do reach better approximations. And that's what we continue to do.
Quoting Wayfarer
Even Schopenhauer, someone who one might think would disagree that we cannot know the thing in itself says:
"Meanwhile it is carefully to be noted, and I have always kept it in mind, that even the inward observation we have of our own will still does not by any means furnish an exhaustive and adequate knowledge of the thing in itself. For even in self-consciousness, the I is not absolutely simple, but consists of a knower (intellect) and a known (will); the former is not known and the latter is not knowing, although the two flow together into the consciousness of an I. But on this very account, this I is not intimate with itself through and through, does not shine through so to speak, but is opaque, and therefore remains a riddle to itself."
(WWR V.II p.196)
The above is a mix of the classic translation with a newer one. The meaning is the same in both, however.
Interesting interpretation, I must say. Ill have to think about it, try to find some correspondence with the relevant text.
Agree, but the awareness of will is not an appearance. We may not know [I]what[/i] it is, but that it is, we can have no doubt.
Quoting Manuel
But the uncertainty principle shows that theres a limit to how exact we can be.
Well, if we don't know what it is, how can we say that it is? We could be wrong. But let's be somewhat more permissive:
This is tricky. It depends on how you define "will". It it's the ordinary use of will, such as willed action, this doubtful. If it's Schopenhauer's, then one has to see what merits it has. It's not trivial. I constantly go back between Hume and Schopenhauer here.
I think we can have (almost) no doubt that we have experience. Beyond that, I think we should be careful in ascertaining certainty in almost anything, outside maybe mathematics.
Quoting Wayfarer
Yes. At least, this is what our theories at the moment show, which may indeed hold up to future discoveries.
On the same grounds as Descartes cogito ergo sum: even if you suffered complete amnesia and forgot your identity, you would be aware of your own being. Thats the point about *any* being: on some level it is aware of its distinction from what is other to it. It knows that it is.
Quoting Manuel
Buddhists would say that our grasp of reality is inversely proportional to our degree of attachment. But that belongs to another thread (or forum).
That seems like an over-interpretation, all the cogito says is that in order to doubt you have to think and in order to think you have to be real, thus doubting your own reality results in contradiction. This doesn't automatically imply self-awareness or awareness of being.
Id go with self-consciousness myself, rather than self-awareness.
Self-something, at any rate.
Why is that? I know the term "doubt" is sometimes used to refer to an emotional state, but here I think it just means demanding justification for a proposition.
Yep, pretty much where Im coming from. Self-awareness implies sensibility; self-consciousness implies logical thought. Doubt, insofar as it is a relative judgement, presupposes logical thought, of which the subject himself must be conscious. If such be the case, then we can just say self-consciousness represents the entirety of that which the subject himself must be conscious, from which follows the notion of a strict requirement, or what can be termed a principle.
Of having experience yes. On being able to distinguish between willed action and mere reflex, also. On some metaphysical postulate about some blind drive the universe follows (as well as us), that's further steps more advanced than experiencing or "willing" (in the common usage of the term).
Quoting Wayfarer
And they may be right. This doesn't get us closer to the thing in itself. At least, I don't see how at the moment.
This isn't obvious to me at all. I see doubt only as a kind of abstract epistemological "move", similar to a move in chess. It isn't something inherently mental.
Thats not something I postulate, and something that I question in Schopenhauer; Im much more drawn to the idea aspect of his philosophy, than the world as will aspect, which I'm frankly sceptical of. (Actually, coming to think of it, I much prefer the Hegelian geist, but never mind.)
Incidentally I went to an open-air social gathering yesterday, a out-door Philosophical Symposium, the subject of which was 'The Suffering of the World: Schopenhauer's Christian Buddhism.' I hadn't attended before - it was held in a park near Sydney Harbour, convened by an informal group organised by the main speaker (picture below under the Peroni sign.)
I felt the actual lecture concentrated too much on the familiar 'Schopenhauer as pessimist' meme, and not at all on the idealist side of his philosophy, but never mind, it was enjoyable to sit around a table and talk about philosophy with actual people. :wink:
As far as metaphysical ideas go, his is not bad. But it has problems which you also feel. Obviously a very hard topic to talk about in general.
Quoting Wayfarer
Very cool! You have no idea how jealous I am that I've never been to Sydney. My dream city to visit. Very interesting topic to discuss and seeing such a crowd attending is just fantastic.
Quoting Wayfarer
Yes, personal dynamics alter the conversation quite a lot. Being able to emphasize or make facial or give certain looks can be persuasive and enriching. Nice to be able to find such groups.
As for the speaker emphasizing pessimism, yeah, it's a problem with Schopenhauer, it's easier to talk about than his version of transcendental idealism, which can be either hard to get across, or if people are told, they don't (I think, at least in my experience) realize the implications such views have, which are just radical.
I suppose one needs a bit of the philosopher gene to be utterly dumbstruck by what others take to be too obvious to even mention. On the other hand, the more variety, the better I suppose.
Thanks for sharing! :cool:
I put it to those I was talking to, that this is simply a description of egoic consciousness (something Im well acquainted with), that is always seeking pleasure or satisfaction in the sensory domain. I remember a talk from a Tibetan Lama decades ago, where he made the same point, referring to Mick Jagger singing I cant get no satisfaction. Its the nature of pleasure, the hedonic treadmill.
Schopenhauer seems to think it can only be ameliorated by the sublimity of high art or a severe asceticism (which so far as we know he never practiced.) But life has its simple pleasures too, and - something Schopenhauer does acknowledge - there is also an enduring satisfaction in being empathetic to the suffering of others. As always, Ill do more reading, but I do think Schopenhauers pessimism was lacking in that regard.
:up: I think this requires qualities of character. Notably, poise. And the appropriate orientation or attitude. The society we live in is such that it doesn't recognise those qualities, rather it relies on continual stimulation to incite cravings and consumption, hence we all become 'consumers'. Being 'happy without reason' entails throwing that off, which is not necessarily an easy thing to do.
My feeling is, Schopenhauer did not, himself, cultivate that kind of personal discipline - not wanting to be overly judgemental, as I'm no paragon. But I suspect that the kind of life he lead, didn't really lend itself to attaining any kind of real poise or equanimity, which is why he wrote so much about being disturbed by boredom and craving.
[quote=Sri Krishna]But those who overcome the impulses of lust and anger which arise in the body are made whole and live in joy. They find their joy, their rest, and their light completely within themselves.[/quote]
[quote=Wikipedia,Yamas;https://en.wikipedia.org/wiki/Yamas]The yamas (Sanskrit: ??, romanized: yama), and their complement, the niyamas, represent a series of "right living" or ethical rules within Yoga philosophy. The word yama means "reining in" or "control". They are restraints for proper conduct given in the Vedas and the Yoga Sutras as moral imperatives, commandments, rules or goals. The yamas are a "don't"s list of self-restraints, typically representing commitments that affect one's relations with others and self. The complementary niyamas represent the "do"s. Together yamas and niyamas are personal obligations to live well.[/quote]
Apparently, he did do quite many things that brought him joy, walking his dogs, eating sausages in a tavern, going to the theatre and listening to music, and lots of other small details along this line.
You're right about the asceticism part; he never took it that far. But I suppose he just had some form of depression. It's not that his pessimism per se is wrong, one can view the world that way, but it's a particularly gloomy way of looking at the world, which is not necessary.
But given the state of the world, maybe his pessimism was in many respects, severely understated. We are running towards extinction, with enthusiasm.
Still, life's small pleasures, empathy and of course the arts, are now more important than ever. Keep looking at those majestic mountains.
That's true, And yet:-
Not only does he distinguish between - let's call them - real appearances - and - "chimeras" - unreal appearances but he also allows the existence of something beyond or behind appearances. .
He is so embarrased by this that he includes an awkward and unconvincing shuffle to disguise the fact:-
He uses "notion" to designate abstract ideas, of which he denies the reality. So his use of the term here is puzzling.
It seems to me that while appearances are, indeed, all that appears to us, that the distinction between reality and appearance is available to us, not only within appearances, but also behind or beyond them.
Quoting Patterner
I like all your quotations. But don't they also reveal that happiness (and therefore also unhappiness) is complicated? I understand the implication to be that there is no one right approach to happiness/unhappiness.
I'm fond of Wittgenstein's comment in the Tractatus:-
At face value, that's nonsense, of course. The same person living in the same world may be happy at some times and not happy at other times. Neither is necessarily a permanent state. But I think the meaning is that happiness and unhappiness affect how one interprets everything in the one world. "Glass half full" and "Glass half empty" are not about different glasses, but different perspectives on the same glass. Happiness and unhappiness affect how one interprets everything.
There's another point, though, Quoting Manuel
Doesn't this show that happiness and unhappiness are not necessarily mutually exclusive?
Quoting Manuel
I agree with you that the world seems in a particularly bad way at the moment, There are many good reasons for being fearful, even alarmed, about the state of the world order these days. But one may reflect that it is not unusual for there to be good grounds for fear and one's worst fears may well turn out to be excessive. (Most of my childhood and youth was overshadowed by the threat of a nuclear holocaust.)
But there is a counterbalancing excess of optimism, which loses touch with reality and sails gaily and blindly into disaster. We need balance and realism to function successfully in the world. Schopenhauer's (perhaps only official) totalizing gloom is a mistake, but the radical optimisim of Dr. Pangloss in Candide is no better.
I think you misunderstand, "the world" is as interpreted. Therefore the world of the happy person is a completely different world from from the world of the unhappy person, and a difference of interpretation is irrelevant because interpretation is already integral to "the world". That difference is therefore a difference in the world. This is due to the role of the subconscious in interpretation.
If the happy person and the unhappy person are the very same person at a different time, then the world has changed for that person. If you review some of the psychological approaches to depression you'll probably understand this better. Happiness and unhappiness are the result of how one interprets things, so the movement of one to the other is a change of interpretation, the mode of interpretation being logically prior to "the world", as interpreted. Psychological disorders in general, must be understood in this way, because "the real world" is always as it appears to the person, not something separate and independent. Psychosis is not treated by getting the person to understand that what they experience is not the real world.
Yes. The irony or strange aspect about this here is that Schopenhauer does not mention much these happy moments, choosing to speak about art, which is fine and important. But it may overplay his pessimism.
Quoting Ludwig V
That's a good point. Today we add climate change and even less controls than before on the nuclear issue. As Bertrand Russell pointed out: "You may reasonably expect a man to walk a tightrope safely for ten minutes; it would be unreasonable to do so without accident for two hundred years."
One day the boy who cried wolf will be right. Hopefully not soon, but, sobriety ought to make us see we are not doing good as a species at all. It could change, absolutely. But it's yet to happen.
Quoting Ludwig V
This framing is fine. I do think something like us being "indirectly" aware of whatever IS mind independently is probably what we do. The exact mechanism involved in this will depend on your own philosophy.
It's still an important step removed from direct access.
I don't know enough about Berkley to know his influences (I read him pretty much blind), but this actually makes a lot of sense if one looks at his philosophy as essentially recapitulating the "classical metaphysical tradition"*, just through a sort of bizzarro world, fun house mirror setting of modernity.
IMHO though, it ends up looking terribly deflated.
*The term "classical tradition" often gets employed to name the wide, but surprisingly unified blend of metaphysics that dominated from late-antiquity to the late medieval period in Pagan, Jewish, Islamic, and Christian thought (broadly a synthesis of Plato, Aristotle, Neo-Platonism, and Stoicism, with different elements emphasized).
Thanks for producing those particular paragraphs, as it toucheth on something ( ;-) ) which a neo-thomist such as Edward Feser would say is radical shortcoming in the Bishop's philosophy.
It is that Berkeley collapses the distinction between sensation, imagination, and intellectual abstraction (for example in this post by Feser). In Aristotelian-Thomistic philosophy, intellect plays a regulative role, actively grasping universal Forms rather than just passively receiving impressions. Without this distinction, Berkeleys account of knowledge risks reducing all cognition to subjective perceptions, which an Aristotelian would find inadequate - his "ideas" seem to be more like what the Thomist would designate phantasms (mental images) rather than concepts grasped by intellect. As a result, his theory of knowledge risks reducing rational thought to subjective perception rather than a proper engagement with reality. So, a Thomist critic might say, 'Yes, reality depends on God, but not in the way Berkeley imagines. The worlds intelligibility does not arise because God perceives itit arises because God has endowed it with form and finality, which reason can grasp independently of sensory perception.'
@Count Timothy von Icarus, @Leontiskos
I have a lot of trouble with the term "world". It gets used of the worlds of chess and football and physics, of the "lived world", of the different worlds that orbit the sun and who knows what else? There's nothing wrong with your interpretation of it. You are also right that in this context the interpretation involved is a bit mysterious, because it is not the result of a conscious process.
But I did think that it was appropriate to take "world" in the context of the TLP as single and unique, because of the opening remarks:-
[quote"TLP"]The world is everything that is the case.
1.1The world is the totality of facts, not of things.[/quote]
Plus, even when, later on, he deals with the interpretations we make of the world, he about "seeing as.." which suggests to me that he is still thinking of a single reality interpreted in different ways.
Quoting Metaphysician Undercover
I think this needs to be put somewhat differently. For me, "the world has changed for that person" suggests that person is living in one world, which has changed. I would suggest something like "then that person has changed from one world to another. But perhaps that would perhaps raise questions about whether that person is the same person.
I'm not quarrelling with the point that happiness and unhappiness affect how we see everything. So these moods are not simply conditioned by the way the world is. But it is complicated, because sometimes the way the world is can change our mood. I would suggest that it is a question of interaction with the world, not a one-way street.
Quoting Metaphysician Undercover
I'm no expert, but I do understand that you don't deal with psychosis by presenting evidence. Psychosis is not unique in this. It is also a mistake to think that religious beliefs can be dealt with by presenting evidence; it is not a matter of evidence, but of how one interprets the evidence. But if a non-psychotic person can treat a psychotic person, doesn't that suggest that, at some level, they are both living in the same world?
Art can make people happy. Perhaps he just thinks that art is more appropriate to a philosophical context than the pleasures of walking the dog.
Quoting Manuel
I didn't mean to imply that the nuclear threat has disappeared. On the contrary, it may be more serious now than it was in the last century. It has just been superseded by the (possibly more serious) threat of climate change. My expectation is that it will be dealt with. But the process will be messy and only partially effective.
Quoting Manuel
Indirect access to reality is still access to reality. I suppose that introspection counts as direct access? But there, the distinction between reality and appearance collapses.
Yes. The fascination, for me, is tracking the distortions and errors that allow him to reach his conclusions. It's a long list. And yet, he somehow manages to put his finger on ideas that are not simply a recapitulation of the classical tradition and which we are still arguing about. I mean that (on my understanding), he is the originator of what we now know as the idealist tradition in philosophy; also, I don't know of earlier philosophy who explicitly argues for relativism about all our empirical knowledge of the world.
There is one topic that you may not know about, but which, I think, is helpful in understanding Berkeley. Have a look at SEP - Occasionalism
A side-note on Dr. Johnson. I doubt if Dr. Johnson thought he was making an argument against Berkeley. He must have intended his gesture as a counter-example. At one level, Berkeley undoubtedly has a reply. He needs to "translate" "Dr. Johnson kicked the stone" into a redescription in terms of ideas, in the same sort of way that he would translate an apple falling on the stone into ideas. So far, so good. But in this case, he has to deal with God's ideas and Dr. Johnson's ideas and their interaction. The SEP - Berkeley (3.2.6 Spirits and Causation) says:-
So it isn't at all clear how the translation would work in Dr. Johnson's case.
PS added later.
I don't think this is a marginal issue. Tallis' article proposes a different conception - the mind as embodied and enactive. I'm very attracted to this approach and not just in relation to Berkeley. But he does not identify what failings in Berkekely's doctrines might make this approach more plausible. If I were constructing such an argument, it would be at this point that I would intervene.
You are right that the quotation fails to distinguish between sensation, imagination and intellect. I would attribute this to his empiricist approach to philosophy, especially to the doctrine that all our knowledge comes from the senses; so it goes deeper than just Berkeley. I read him as based on Cartesian Dualism but adapting it, with the result that God is placed as the unseen reality moving the world rather than matter. His ideas about God, I understand, are derived from Malebranche's occasionalism. He doesn't seem to refer much to Thomist or Aristotelian doctrines, though he does re-introduce the concept of final causes, which had, of course, been abandoned by the new physics.
I don't know about Thomism in enough detail to respond to that alternative approach in detail, though I think I can see the sense in it.
I believe this is what Hume does as well, so it must have been a trend at that time. Hume seems to reduce ideas, to sense impressions as the fundamental base of an idea. Then all ideas, even complex ideas, become like a compilation of sense impressions. This effectively evades the issue of universals.
Quoting Ludwig V
The issue here is the conclusion that the person is living "in" the world. Why do you conclude that the person is in the world rather than concluding that the world is in the person? The changes in "the world" which we were talking about are changes which are caused by the person changing from being happy to unhappy. The unhappy person perceives the world in a different way from when that same person was happy. If these are changes to "the world" for a person, don't we have to conclude that the world is the perception, and the world is within the person, not vise versa?
Quoting Ludwig V
The issue is where do we position "the world", in this interaction. Like Kant shows, the world is better positioned as phenomenal, how things appear through sensation, rather than as the separate thing itself. The reason for this separation is that mistakes inhere within the appearance, as mental illness demonstrates. If we do not allow for this separation then there is no way to account for the mistakes which the sense apparatus makes, in presenting its representation to the conscious mind. And since the mind only has the appearance to base its judgement on, it must allow for the logical possibility that the sense apparatus is completely mistaken, in an absolute way, as the skeptical starting point. This necessitates, as a starting point, that "the world" refers to the appearance, not the independent thing itself.
Quoting Ludwig V
I don't see how that conclusion would follow. We must allow that "world" is a defined term. Your example shows interaction between two people. Therefore we need a definition of "world" which implies that if two people interact they share a world. But by the principles above, my world is how things appear to me, and yours is how things appear to you, so interaction appears to be simply a matter of two worlds interacting. By what principles do you reduce two distinct worlds interacting into one united world? You can define "world" as that unity, for that very purpose, but that's a matter of begging the question. Furthermore, it puts all those mistakes discussed above into some sort of limbo, where in one sense they have to be part of the world, but in another sense they have to be excluded from the world. Therefore that sort of begging the question, with a definition designed to evade the issue, only produces more problems.
I hope you are right, but I see reasons for skepticism.
Quoting Ludwig V
Introspection is limited, we don't know how our ideas arise, nor do we know how thoughts connect to one another. It is a mistake in modern philosophy of mind to believe that everything (or almost everything) must be accessible to consciousness. It's not.
But you do have a good point, in so far as we are going to speak of the "in itself", I do believe that we know some fundamental aspects about mental reality in merely having consciousness.
It's not clear to me how much this says about anything else about the world at large.
Right! As Ludwig says
Quoting Ludwig V
That's the source of it.
Quoting Ludwig V
I'm no scholar of Thomism, but I've got a grasp of the basic outlines of what Edward Feser (who's a good source in these matters) calls 'Aristotelian-Thomist' (A-T) philosophy - Aristotle's matter-form philosophy. I also read a little of Jacques Maritain, who was hugely influential in the Catholic left in the mid 20th c. and a profound philosopher. From whom The Cultural Impact of Empiricism:
Hume and Berkeley are both nominalists. Nominalism is one solution to the issue of universals. There are others. But I don't see how accepting one solution to the problem is evading it.
Postscript - afterthought
Quoting Metaphysician Undercover
If our world is the totality of our perceptions, given that the perceiver of a perception is not perceived in the perception, perhaps we need to say not the world is within us, but that we are our world. I could live with that.
Quoting Metaphysician Undercover
I understand my world to be everything that I interact with, together with myself. I interact with many different kinds of thing, some of which don't have a location in any normal sense. Perceptions are one example of this. So I'm not clear what the question is asking for.
Quoting Metaphysician Undercover
I agree with that. But consider - if all you have to go on is appearances, how do you know when you have made a mistake?
Quoting Metaphysician Undercover
Some appearances are mistakes. Some appearances aren't mistakes. It would be a mistake to think otherwise. The question is how to tell one from the other.
Quoting Metaphysician Undercover
My world is what I interact with. Your world is what you interact with. It follows that if I interact with you, you are a part of my world, and that if you interact with me, I am part of your world. I don't say those two worlds are identical. I do say that they overlap.
Quoting Metaphysician Undercover
Macbeth's delusional dagger is, in one sense, part of Macbeth's world. But since it does not exist, it is also not part of his world.
The issue, is that we tend to extend "the world" beyond the limits of our interactions. That is what makes us want to say "I am in the world", in stead of "I am the world". The world is larger than me, so I am in some sense a part of the world, and that's what inclines me to say that I am "in" the word. The problem is that this extension of "the world", to include things beyond the limits of my interactions, requires principles, and the principles cannot be directly derived from the interactions, as they are principles of extrapolation.
Quoting Ludwig V
Given what I said above, to deny the distinction between the world as directly perceived through sensation (one's interactions), and the extended world (the world which consists of more than one's interactions), would be to eliminate the need for principles derived from other than direct sense perception. Therefore it is an evasion of the issue.
Quoting Ludwig V
This is a very good question, and it points to the reason why ontology, or metaphysics in general, is a very difficult subject of study. The question ought not be taken lightly, as many do. The issue can be represented by example as:
"How do we know when empirical science is mistaken, if empirical science is consistent with sense observation?".
The common approach is to evade the problem (explained above), deny that science utilizes principles of extrapolation which are derived from somewhere other than empirical observation, and insist that pure, well-principled science cannot be mistaken. Hume demonstrated this problem of extrapolation as the problem of induction. And when we recognize that such nonempirical principles must be applied, then we can broaden that category, of nonempirical knowledge to include the principles by which we interpret sensations. And then it becomes apparent that such principles are necessarily prior to any coherent sensation, as what makes sensations intelligible in the first place.
So this is revealed as the fundamental, basic, or foundational mistake, the idea that knowledge follows from sensation, instead of the reality that sensation follows from knowledge. Then empirical knowledge is understood as an enhancement, or branch of a larger underlying body of knowledge. These are the levels of distinct potentiality described by Aristotle. Prior to being educated in science, a person has the potential to be educated in this way, and after being educated the person has developed to a new level, a different potential.
To answer your question then, "to know when you have made a mistake", requires being educated in fields which are distinct from those fields which focus on analyzing sense appearances. The most inclusive (broad) field here is morality. In morality we learn about what we ought to do, and ought not do, and the basic precepts of what it means to make a mistake. This is the base level for "acquired knowledge".
Quoting Ludwig V
The basic point to understand, which can be derived from what I called the "foundational mistake", is that all appearances have inherent within them, mistakes. This is the difference between how the thing is in itself, and how it appears to a person. To recognize, and enforce this differentiation, Aristotle imposed the law of identity. This law recognizes that the identity of a thing is within the thing itself, rather than what we say about the thing. This principle recognizes the separation between the thing and how the thing appears, and acknowledges mistakes within the appearance, through the difference between essences and accidents.
Quoting Ludwig V
In this way, we have a multitude of worlds. We could produce a model, a representation in which we have up to an infinity of worlds overlapping, but that becomes extremely complex. However, it is intuitive to believe that there is one independent world, and all these worlds are really just part of one united world. Producing this model, or representation, requires that we adopt principles which are derived from somewhere other than our own personal interactions. We have an example of these principles, as the basic inherited knowledge, or potential, which we are born with, and then the basic moral knowledge which builds from this instinct, producing that conception of unity which is so intuitive to us. Hume dwelled on these differences, between distinct sensations or ideas, and the continuity of a united experience.
What are you guys calling appearance? The context of your dialogue obtains from Kants notion of intuition with respect to the world, in which appearance has nothing to do with mistakes, and, the world has nothing to do with appearances.
The only way your dialogue works, is to correlate appearance with looks like, while Kantian phenomenal correlation is with respect to presence of. In order for your arguments to hold, therefore, re: mistakes are inherent in appearances, you have to allow the mere presence of a perceived thing a form of cognitive power, or, grant to appearance more content than the space and time Kantian doctrine permits.
Not to curtail your dialogue, but as stated its not consistent with the reference upon which it is, at least initially, premised.
Thing is .Im sure both of you are fully aware mistakes in empirical cognitions inhere in judgement, not in appearances. And mental illness is not the rule, but the exception to it.
I agree that we know that there are things that we know that are not available to the direct access of introspection. If we know that, we have access to at least one fact about them - that they exist. If we know that we must have indirect acess to them.
Quoting Manuel
If we knew that there are some things that are not available to consciousness, they must be available to conciousness.
Quoting Manuel
How do we distinguish between mental reality and other kinds (such as physical or abstract reality) unless we have access to those other kinds?
It seems that we have a similar level of knowledge about those ideas. That helps.
Put it this way. For me, perception requres understanding. Without that, one only has a "raw sensation" which is meaningless.
This is just behaviourism restricted, for some reason, to animals. But many people were quite happy to explain human beings in that way as well as animals. It is a way of thinking about them, not vulnerable to a simple refutation. (Compare religious belief).
But If "a dog reacts in a similar manner to the similar particular impressions his eyes or his nose receive from this thing we call a piece of sugar or this thing we call an intruder;" how does it not see or smell the sugar or the intruder and know perfectly well what they are - what the appropriate reaction is?. I'm bewildered.
If he has not the idea or concept, he does not know the thing. But since he responds appropriately to the thing, he has a concept of it. Not necessarily the same as yours and mine, but similar.
A couple of metaphors do not clarify anything.
I do agree that our discussion is messy. That's partly because the context is a bit messy. From my point of view there is more than one context. There's Berkeley and Schopenhauer, as well as Kant. The immediate spark, for me at least, was the idea that happiness and unhappiness affect how we experience or interpret the world, or the phenomena or appearances. My problem with the Kantian system is simply that the idea of the noumenon. I understand this as meaning a something-or-other that sits "behind" or "beyond" the phenomena" and which cannot be known. I'm not a fan.
Quoting Mww
I don't think that we first recognize that something appears to us and we then make judgements about it, or rather, even to recognize that something has appeared is a judgement. Judgement is always included in every perception.
Yes, mental illness is not the norm. There are many different kinds of problem here. The common element is the issue of how one interprets the world and how that process is not an abstract issue of knowledge, but is conditioned by all sorts of other factors.
We discussed this at length on that thread on rationality in humans and animals. I distinctly recall holding the minority view in that thread, as I maintained that the Aristotelian distinction of h.sapiens as 'the rational animal' is a valid ontological distinction. In other words that h.sapiens and canids (etc) are beings of different kinds. I said that the ability to speak, count, create technology, pursue science, and the like, amounts to a difference in kind, not simply one of degree.
Now as for whether that is a 'religious belief'. The reluctance to acknowledge what I take to be a clear ontological distinction between homo sapiens and other species seems to stem from a broader philosophical commitmentone shaped by the widespread influence of Darwinian naturalism on our conception of human nature. In many ways, evolutionary biology has reinforced the view that we are fundamentally continuous with the rest of nature, which is something many cherish, both as an expression of scientific understanding and as a foundation for ecological and ethical values. And I do not in any way wish to diminish the importance of those values.
However, I believe this perspective risks overlooking a real distinction that has profound existential implications. Our capacity for self-awareness, symbolic language, and complex tool-use sets us apart in ways that are not merely matters of degree but of kind. While we should certainly recognize our biological continuity with the rest of nature, we should not let that recognition obscure the radical difference that defines our cognitive and cultural life. I suspect that some resistance to acknowledging this distinction arises because evolutionary theory has, in some sense, come to function as a meta-narrativea way of understanding our place in the cosmos that, in its broadest cultural expression, tends to downplay discontinuities in favor of an overarching unity by flattening such ontological distinctions. It is one of the consequences of the cultural impact of empiricism that Jacques Maritain (and, in a different way, George Berkeley, are criticizing.)
Quoting Ludwig V
The point of Maritain's essay is precisely that a dog (or other non-rational sentient beings) lack the specifically human capacity to form concepts. 'The human intellect grasps' says Maritain, 'first in a most indeterminate manner, then more and more distinctly, certain sets of intelligible features -- that is, natures, say, the human nature -- which exist in the real as identical with individuals, with Peter or John for instance, but which are universal in the mind and presented to it as universal objects, positively one (within the mind) and common to an infinity of singular things (in the real).' This is, of course, Aristotelian realism, and Maritain says he is an Aristotelian.
The Edward Feser blog I posted puts it like this:
I would say 'reason' rather than 'thought' but I think the point is clear. So, no, I don't think that dogs and cats entertain concepts, but I hold that on philosophical, not religious, grounds. And that this goes against the grain of modern culture, precisely because of the cultural impact of empiricism.
I don't strictly adhere to Kantian principles, so I left Kant on the last post or two. I grant to appearance, more content.
Quoting Mww
I referred to Kant more as an example, than as a premise.
Quoting Mww
I believe that since appearances are the creation of the living system (sense apparatus, I think I said), there is nothing wrong with asserting that mistakes inhere within appearance. This is the position Plato took, the senses deceive us. And, good evidence of this is hallucinations and mental illness.
If such were the case, though, youd have a logically consistent answer regarding when a mistake is known to have been made.
Quoting Ludwig V
Dont blame ya; Im not either, but probably for different reasons.
-
Quoting Metaphysician Undercover
Thats fine; Im not going to argue with that. Myself, I prefer to think of appearance as something that happens to, rather than being a creation of, the living system.
That's a simple difference of opinion. But it's actually very significant in metaphysical implications.
Rather than fear of religion or the cultural hegemony of materialism, I think your biggest problem is that you think that your view is obvious. I think you should put more energy into making a positive case for the ontological distinction you're introducing to other people, rather than attacking perceived obstacles (darwinism, modernism, etc.) as if that alone were enough. After all, what is "clear" to you might be completely counterintuitive to others.
Thanks. It is a theme I write a lot about. The post you're referring to is a follow up on a previous conversation in another thread on rational thinking in animals and humans. I don't think my view is obvious, I've done a lot of research, reading and thinking about it over a long period. I'm criticizing a view which is thought obvious by many.
If you want me to try and explain the reasoning in more detail I'm more than happy although it is probably more relevant in that other thread.
When I think of a critique of empiricism I think of Kant. He criticized the idea of tabula rasa that persisted in empiricism. Hence his whole philosophy concerning the active position of the mind with respect to what we perceive.
Kant even introduced the forms of sensibility (space and time) as transcendental forms that apply to external things.
In the second edition of the Critique of Pure Reason (1787), Kant introduces the "Refutation of Idealism", where he argues that the existence of the external world is not just probable but necessary for self-consciousness.
Kant argues that the existence of external objects is a necessary condition for self-consciousness. His reasoning follows these steps:
1. We are aware of our own existence in time. We experience a continuity of thoughts and changes in our mental states.
2. To be aware of time, we need an objective reference point. Time is not something we perceive directly; we only understand it in relation to external events.
3. These external events must be stable and distinct from our minds. If only internal perceptions existed, we would have no fixed framework for organizing our experiences in time.
4. Therefore, the existence of an external world is necessary for self-consciousness.
This argument is based on the idea that we cannot be aware of ourselves without an external reference frame. The external world is not just an optional assumption. it is a prerequisite for our experience of the self to make sense.
Kant does not claim that we know things as they are in themselves (noumena), but he does assert that something external structures our experiences.
You are right, of course. It's probably not a good idea to re-litigate all that here. Briefly, I don't know what the difference is between an ontological distinction and any other kind, so forgive me if I just talk about a distinction (or difference). It seems to me that there are differences between h. sapiens and other creatures and similarities. A big part of the issue is which of them matter, and that depends on the context. I object to emphasizing the difference and then thinking that animals do not experience pain in much the same way as we do. But it is easy to push the similarities too far and then applying inappropriate moral values to them. It's a question of balance and context and of attention to the details of each case. Dogs are a special case because of the relationships that they have which human, which are not unparalleled but are extreme on the spectrum of human/animal relationships.
On the business about intellect, imagination, and sensation, to treat these as entirely distinct abilities, each functioning in its own box may seem clear, but distorts the complexity of our cognitive capacity and grossly neglects the importance of our not merely existing but acting in the world. This is a very large subject. It would be better, perhaps to take it to another thread.
Quoting JuanZu
I can follow the line of thought until this point. But this is where, for me, it falls apart.
I'm really surprised to hear you say that, I've seen you explicitly invoke obviousness several times to defend your views. Here is a very clear example:
Quoting Wayfarer
Although, you wrote it a long time ago. Maybe you've since changed your mind?
Perhaps. In Kantian metaphysics, though, the notion of appearance is merely intended to grant ontology in general, which serves to limit metaphysics to the conditions of a logical science, entirely internal to the human intellect. Which reduces to .whatevers out there is whatever it is; all that remains is to expose how the human intellect of a specific dedicated form treats it.
Can I ask, what defines this difference for you, that is the difference between a difference in kind and a difference of degree. I would say, for example, that a difference in kind constitutes a complete break with no possibility of continuity between the two, where as a difference of degree implies a continuity. To exemplify, I would say that the differences between the perceptions of the different senses can be differences of kind, so that the object of sight, and the object of hearing are different kinds, while differences within one sense, like the difference between blue and red, are differences of degree. The latter implying a continuity between the two, the former a discontinuity.
My opinion is that we have to be very cautious in our judgements concerning this subject because sometimes a very large jump of degree appears like a difference of kind. So for example, nonvisual electromagnetic activity appears to be a difference of kind, because it is not visible, but it has really been shown to be a difference of degree. Conversely, assuming that differences of kind can be reduced to differences of degree produces faulty conceptions like the philosopher's stone, and prime matter. So we approach the ancient question of how many different ontological kinds are there.
I think we can see, that living beings have developed (through evolution?) capacities of different kinds. For instance, hearing detects vibrations of molecules, while taste and smell (both perhaps of the same kind) detect changes to the molecules themselves, physical or chemical interactions with the senses. We could say that these are different kinds of activities being detected. And all of this presents us with a bit of a philosophical puzzle in itself, how does a capacity of a different kind come into existence in evolving life forms.
So if we judge that intellection, the capacity to reason with abstract conceptions, is a difference of kind, I think we need to justify this judgement. Now we might refer to "the object" of this capacity, like I referred to the difference between the object of sight and the object of hearing, as what the capacity is working with. I can think of a number of possibilities. We might say abstract concepts are the object of intellection, but this is problematic because they do not really qualify as "objects" by the law of identity. Then we have signs or symbols, as the possible object, but this is equally problematic because signs get reduced to anything which carries meaning, so that all sorts of creatures can be seen to observe the meaning of signs. Another possibility is "object" in the sense of a goal, intention or final cause. Is it the case that recognizing goals, as final cause, and the whole structure of moral philosophy which developed from this, is what constitutes the object of intellection? This would be what Plato called "the good". But again, the question is can we have such an object, which satisfies the requirement of the law of identity, and if not, the whole presumed activity is invalidated as fictional. This is the issue of objective morality.
To summarize the issue, to judge the power of intellection which human beings have, as a capacity which constitutes a difference of kind, from the other capacities which other creatures have, rather than a difference of degree, requires justification. Justification requires that we describe what that capacity does, in its actuality, and this means that we look at the movements of its objects. Before we describe the movement of the objects, we need to identify those objects, because falsely identifying objects will result in a false description, and a faulty justification.
Quoting Wayfarer
The problem, is that claiming there is a radical difference, and that we need to recognize this difference as a radical difference, because it has serious ontological implications, does not itself, really qualify as "recognizing the difference". It is a matter of asserting a difference without actually recognizing it. Without the capacity to describe, or somehow demonstrate that difference, it's just a blind assertion. And madmen can make many such blind assertions about many crazy things, but unless they demonstrate something, we just dismiss them as crazy. Further, as Berkeley has exemplified to us, if what you demonstrate is contrary to what people already believe, the demonstration must be carried out on their terms, and that makes it even more difficult to demonstrate what you assert.
Quoting Mww
So the problem I see, is that this assumes a sort of Cartesian separation between external and internal. But, if we are to accept this separation as real, we need to determine a boundary, and this is where the problem lies. If we assume a purely internal, "within the mind", and a purely external "out there", and this is what the Kantian distinction forces on us by making the "out there" completely separate, then we have to account for the reality of the sense organs as somewhere in between.
This is very problematic, because the boundary by Kant's description must be a complete separation, allowing nothing from "out there" to be internal. So if the senses are causally affected by activity which is external, they must be completely "out there" themselves. But if the senses are creating the phenomenal appearances in the mind, they must be completely internal. So the Kantian system is really inadequate to account for reality because it doesn't allow that the senses partake of both, the external and the internal. And the Kantian system is caught by the "interaction problem". That's the problem with naming the noumenal as completely inaccessible to the human mind.
If we discover them. If not, trivially, they remain in the dark.
Quoting Ludwig V
That's almost a panpsychist claim, that everything is experience-realizing or experience-involving. That's not clear. We know that animals have sensations we cannot experience, like dogs will smell or squid with vision, etc.
You could say that since we know about it, it also involves our consciousness. But that's not the same as us experiencing it, which I take to be the important part of having consciousness.
Lastly, unless we are an evolutionary miracle such that we so happened to evolve to experience everything and know everything, it logically follows, that there are things we cannot understand or even experience, it is beyond our capacity to "latch on to", as it were.
Quoting Ludwig V
Mental reality refers to those things that appear to our minds which need not have a corresponding object of which the mind is referring to.
Physical reality refers to those things that exist absent us, but which we can experience as well.
Mathematical reality is about taking mathematics to be existing entities or things which exist somehow.
And on and on. This is a matter of emphasis; these are not metaphysical distinctions.
Quoting Metaphysician Undercover
You called it a Kantian distinction, which I think much more the case than separation. It is inescapable that the human sensory apparatuses are affected by things appearing to them, which tends to negate the premise the senses and that which is sensed are separated on all accounts.
In addition, it is equally inescapable, hence trivially obvious, that the real physical things out there are not the representational things of experience.
So, yes, a decidedly refined sort of Cartesian dualism.
-
Quoting Metaphysician Undercover
I hesitate to admit the senses are causally affected, but rather think they are functionally affected, in accordance with the natural physiology, which makes explicit they are out there themselves, in relation to the cognitive system itself. That is to say, the sensory devices are just as much real objects as are basketballs and snowflakes.
Quoting Metaphysician Undercover
Ahhhh, but they do not; the senses do nothing but forward information in the form of sensation, again, in accordance with respective physiology. Not hard to understand the senses as merely a bridge between the real and the representation of the real. Phenomena belong to intuition, which is a whole nuther deal than appearance/sensation, which might ..very loosely .be deemed the source of the internal images of the external things.
Quoting Metaphysician Undercover
As stated above, the account does allow the senses to, maybe not partake in so much as distinguish between, the external and the internal.
-
Quoting Metaphysician Undercover
It cannot be completely inaccessible. If noumena were inaccessible to the mind there could be no conception of it. Which highlights a misconception: Kants is a system in which different faculties function in unison. Mind may be understood as the composite of those faculties, but it remains that each faculty does its own job, and when examining the system, to overlay all onto mind misses the entire point of the examination.
Noumena are inaccessible to some faculties but not others, so it cannot be said, or said accurately, they are inaccessible to the mind. Technically, noumena are accessible to the understanding alone, insofar as the understanding is the faculty of conceptions, and a conception is all a noumenon could ever be.
Be advised: you lose absolutely nothing by neglecting noumena entirely when examining human knowledge. The only reason Kant brought it up was to plug an ever-so-abstract logical hole.
(Actually, some secondary literature accuses him of backing himself into a corner, from which his extrication demanded a re-invention of classic terminology, which in turn seemed to demand an apparently outlandish exposition, which really isnt at all.)
Anyway .ever onward.
Quoting Ludwig V
As I've introduced this idea of there being an ontological difference or discontinuity between humans and other animals I will attempt to justify and describe it.
But bear in mind, the origin of the discussion in this particular thread, was in respect of Berkeley's philosophy, and, in particular, his empiricism, and his insistence, with the other empiricist philosophers, that sensory experience is the sole source of knowledge.
As noted in 's response, this contention was criticized by Kant, in an effort to differentiate his 'transcendental idealism' from what he described as Berkeley's 'dogmatic idealism', and also in a more general sense because of Kant's insistence of the fundamental role of the a priori in the understanding. That is an element lacking in Berkeley's philosophy (and indeed in all of the empiricist philosophers).
I introduced what I understand of the Aristotelian-Thomist (A-T) criticism of Berkeley's empiricism, as described by Edward Feser in several online posts. At issue in that criticism is the claim that reason makes use of another faculty, apart from sensation and imagination, and it is this faculty which distinguishes the human intellect. As Jacques Maritain, another A-T philosopher, put it, what distinguishes the human from animal minds, is the ability to grasp universals - the universal 'man' for example. He contrasts that with the intelligence of the dog (and this can be said with all due respect to the status of dogs as companion animals, for whom I have great affection). A dog doesn't form a concept of a class or kind but 'reacts in a similar manner to the similar particular impressions [from] his eyes or his nose'. Feser adds to this, the idea that universals are the fundamental constituents of rational judgement (I will mention again his post Think, McFly, Think, for a more detailed description.)
Getting back to the 'ontological distinction'. Ontology means 'kinds of being'. For instance, in information technology, the ontology of a network would comprise a description of the different kinds of components that it comprises (e.g. servers, printers, routers, pc's etc. Not the inventory, but the kinds of devices - inventory would be separate.) In philosophy, 'ontology' has rather fallen out of favour (as has 'metaphysics'), partially because it's a rather archaic term, but also because it relies on making the kinds of distinctions that sit uncomfortably with naturalism, which tends not to categorise this way.
So - in the Aristotelian-Thomistic tradition, the distinction between humans and animals is not merely one of degreeit is an ontological distinction. Animals are sentient, but their consciousness is bound to immediate experience. Humans, by contrast, possess rational intellect, allowing for abstract thought, self-reflection, and moral reasoning. This places humans in a separate category, traditionally referred to as animal rationale (rational animals) rather than merely sentient beings. It is also the source of what is loosely described by 'the human condition'. I am critical of the way that neo-darwinism, on the popular level, erases this distinction, although that is tangential to this thread. My only purpose in bringing in A-T was to highlight their criticism of Berkeley, which I believe has merit, even though in many other respects, I am prepared to defend Berkeley's idealist philosophy.
I think the problem with this argument has been pointed out to you before: I could make the same case for the pistol shrimp and say that it is an ontologically distinct species because it has the unique faculty of shooting shockwaves out of it's claws. It's easy to prove that humans are different, but you have yet to prove that this is "a difference that makes a difference" as I believe you like to say.
Which is a triviality. The mere fact of uniqueness is not at issue: species are defined in terms of uniqueness. At issue is the capacity to reason, unique to h.sapiens and the entailments of that. So whomever has tried to 'point that out' is mistaken.
Said he, reasoning.
Recall that this was what I originally said was the problem: you argue from the position that your view is intuitive, but when people do not share your intuition you have nothing to say. Basically you can only convince people who in a sense already agree with you.
I can see why that is a problem for the Kantian system. What I don't see is why there is a problem about accepting that, because we have senses, we can interact with our environment in ways that insensate objects cannot; this is one of the markers of being alive. But, of course, in order to establish that one has to persuade people that the phenomena (appearances, ideas, impressions, sense-data) are not a veil between us and our environment, but a window. It's not an easy or straightforward project.
Quoting Mww
Well, that's how we think of them, especially when we have little or no idea how they work. But you seem to ignore the familiar point that the transformation of causal input into information requires a good deal of work.
Quoting Mww
That's a very interesting take - and very helpful, Now I can see that, just as Berkeley, having acknowledged that there must be a cause of those of our ideas that are not under our control, plugs the gap left by his rejection of matter with God, Kant plugs the same gap with noumena. The fundamental problem arises from the idea that our senses do not put us in touch with reality, but separate us from it. Then generations of philosophers wrestle with a problem that is created from the way that the question is asked.
Philosophers get acclimatized to a very general use of words like "appearance". But not all appearances are the same.
For example, Descartes' point that sometimes our senses deceive us is the ground for the philosophically traditional radical scepticism. Escaping from that trap is one of the basic motifs of modern philosophy.
We know that some appearances are misleading. What follows? Does it follow that all appearances are misleading? No. Does it follow that all appearances might be misleading? No, the fact that we can tell that some experiences are misleading means that we can distinguish appearances that are not misleading from those that are.
Quoting Metaphysician Undercover
I can see why that is a problem for the Kantian system. What I don't see is why there is a problem about accepting that, because we have senses, we can interact with our environment in ways that insensate objects cannot; this is one of the markers of being alive. But, of course, in order to establish that one has to persuade people that the phenomena (appearances, ideas, impressions, sense-data) are not a veil between us and our environment, but a window. It's not an easy or straightforward project.
Quoting Mww
Well, that's how we think of them, especially when we have little or no idea how they work. But you seem to ignore the familiar point that the transformation of causal input into information requires a good deal of work.
Quoting Mww
That's a very interesting take - and very helpful, Now I can see that, just as Berkeley, having acknowledged that there must be a cause of those of our ideas that are not under our control, plugs the gap left by his rejection of matter with God, Kant plugs the same gap with noumena. The fundamental problem arises from the idea that our senses do not put us in touch with reality, but separate us from it. Then generations of philosophers wrestle with a problem that is created from the way that the question is asked.
Philosophers get acclimatized to a very general use of words like "appearance". But not all appearances are the same.
For example, Descartes' point that sometimes our senses deceive us is the ground for the philosophically traditional radical scepticism. Escaping from that trap is one of the basic motifs of modern philosophy.
We know that some appearances are misleading. What follows? Does it follow that all appearances are misleading? No. Does it follow that all appearances might be misleading? No, the fact that we can tell that some experiences are misleading means that we can distinguish appearances that are not misleading from those that are.
When Macbeth sees a dagger in front of him, there is no dagger. So we say that there appears (to him) that there is a dagger. Macbeth is deluded. The dagger is a subjective experience - an image, not a reality. When the sun appears over the horizon at dawn, exactly on time as usual, there is no mistake, no delusion. Here, an appearance is perfectly real, perfectly true, perfectly objective. These appearances are revelations, not illusions.
Before Copernicus & co., everyone assumed that it was the sun that was moving and the earth was still. That is a mistake, just like the illusion that one sometimes gets when the train on the next tracks moves in relation to our train. We tend to assume that our train is still because we are still in relation to it and there are no visual clues to tell us otherwise. But this is not a subjective, delusional appearance like Mabeth's dagger. It is just an misinterpretation of the real situation. From a different point of view, the situation would be perfectly clear.
Under certain conditions of sun and rain, rainbows appear. One wants to say that there appears to be a coloured arch floating in the sky. There is no arch. But this is not a delusion like Macbeth's dagger or the relative motion of the trains. It is a perfectly real phenomenon, which is just what it appears to be, except that the physical substrate of the phenomenon is not what we expect; it is millions of rain-drops reflecting the sun's light.
Sweeping up all sensations under one description is misleading and creates unnecessary problems. Look at the details.
The point though, is that if this is the case, then the Kantian distinction cannot be maintained. If the senses are affected by the things sensed, then the senses are noumenal and there will be difficulty keeping the phenomenal as separate from the noumenal.
Quoting Mww
If the senses are out there, and what appears to the mind is in here, then where does the boundary between these two lie? Or, more to the point, how can there even be a boundary? Sensations are just as much "in here" as ideas are "in here", but they appear to be an aspect of the sense organ. If I say that the senses are out there, then the idea of a boundary between in here and out there makes no sense, because the sensations are in here, yet also in the senses, which are out there.
It appears to me, that unless I place the boundary as between the sense and the object sensed, the idea of a separation, or even a distinction, makes not sense. The brain and the senses are all aspects of the same nervous system. Therefore the boundary must be between the mind and the whole body (leaving an interaction problem), or between the senses and the objects sensed. I prefer the latter because it provides a better way to deal with the interaction problem.
Quoting Mww
If you look closely, analyze this described scenario, you will see that you have an interaction problem here. You describe the senses as providing information, but not causing any phenomena. And, you have phenomena as belonging to intuition, a completely different thing from senses providing "internal images of the external things".
By this description, the senses are external in relation to intuition and phenomena, but they somehow create internal images. So we can ask how does that boundary between external and internal get crossed. How is it that the sensations (images) are internal yet the senses are not. And why do you place a further (internal) boundary, or distinction between these images and intuition with phenomena?
Quoting Mww
This capacity, to distinguish between external and internal, which you assign to the senses is an arbitrary judgement. That, distinction is a spatial judgement, so it requires intuition. But you have already separated intuition off, far away. So it's incoherent to say that the senses can make such a distinction, unless we allow intuition to be within sensation itself, and this implies that the conceptual structure needs to be changed to allow the mind to be within the senses.
Quoting Mww
You might say, "to overlay all onto mind misses the entire point of the examination", but I would say that to overlay like this exposes the fundamental fault of the examination. If mind is assumed to be the composite of those faculties, and all the faculties cannot be shown to co-exist as a unity of "mind", then there is an incoherency within the conception.
Quoting Mww
This demonstrates the incoherency of the conception. There appears, "an ever-so-abstract logical hole". The assumption of "noumena" is required to plug the hole. But "noumena" doesn't do anything, as you admit, except create the appearance that the hole has been plugged. Really though, the hole is still there, but it now exists as an abstract object, "noumena", and that abstract object is completely unintelligible. So the whole is filled with a self-contradicting idea, an intelligible object which is unintelligible.
This is analogous to Aquinas' position on God. God is most highly intelligible, but fundamentally not intelligible to the human mind. Aquinas explains this, as God being a pure, separate immaterial Form, independent from matter. The human mind cannot grasp the separate forms due to its dependence on material existence, its unity with matter. This is an attempt to resolve the self-contradiction. Theoretically, after death when the soul leaves its dependence on matter, it can grasp the pure Forms. But the way that Kant turns things around, he appears to make matter the thing which plugs the whole, but that really leaves the hole unpluggable.
Quoting Wayfarer
This is similar to Mww's description of the Kantian conception above, a division into distinct "faculties". What is at issue, is whether the division into these supposed distinct faculties is supported by good ontology, or whether they are just arbitrarily produced for some other purpose (for example, for Kant, an epistemic purpose).
So Kant for instance, denies the relevance of any independent, external existence by designating it "noumena". By this principle, truth as correspondence, is no longer applicable, and this allows Kant to produce all the internal divisions and categories at will, so long as logical consistency, and correspondence with phenomena is maintained.
You can see, that from this perspective, we can simply designate "the ability to grasp universals" as a distinct faculty, and claim this to be a difference of kind. That is why I explained that we need to go beyond this simple stipulation of "distinct faculty", and examine the object of that faculty, in relation to the objects of other faculties. If it turns out that two supposedly "distinct faculties" (with that designation of 'distinct', supporting the claim of a difference in kind) actually have the same type of object, then the claim of a difference in kind is not supported ontologically.
This is what I think is required to support your claim of an ontological difference of kind. If you support it with epistemological categories, and the epistemology is not well grounded in good ontology (like Kant's), then the claim of an ontological difference of kind is not well supported.
True enough, plus, words often get defined in order to accommodate a project. Ive already mentioned the difference in meaning for the word appearance relative to the Kantian metaphysical project, as opposed to the common meaning relative to others.
-
Quoting Ludwig V
These days, there is good evidence for how they work. Before, thus from a metaphysical point of view, how they work wasnt important enough to jeopardize speculative philosophy, insofar as humans are not even conscious of most of that which is under the purview of natural law anyway. Even now, while science has cleared much ignorance, the subject himself in general remains unconscious of most of his own intellectual machinations.
-
Quoting Ludwig V
And how do we tell? From whence does the distinction arise?
Quoting Ludwig V
Not sure what that means. I can describe all sensations as merely that by which I become aware of my environment. I am not mislead and have no unnecessary problems, because the description does not contradict the facts. Someone else, describing senses in some other way, might then think me misled and invoking unnecessary problems, which is fine by me.
Show me the details?
No, that doesnt follow at all. Thats like saying an ice cube is noumenal because it shatters when hit by a hammer.
-
Quoting Metaphysician Undercover
In the faculty of intuition, where that which appears acquires its representation, called phenomenon.
-
Quoting Metaphysician Undercover
Sensations are in the senses? If there were the case, why would we have both? You want the hand to tell you the thing is heavy when all it can do is tell you of the appearance of cellular compression. You want the ear to tell you there is a sound when all it can do is register the appearance of variations in pressure waves. And so on .
-
Quoting Metaphysician Undercover
Senses providing internal images of external things is not what I said.
-
Quoting Metaphysician Undercover
I didnt say all the faculties couldnt co-exist. In fact, I said the mind could be called the composite of all the faculties, which makes explicit they do co-exist. Each faculty can still be imbued with its own dedicated functionality without contradicting the notion of a unity.
-
Quoting Metaphysician Undercover
This little dialectical segment is my fault, for not correcting you here:
Quoting Metaphysician Undercover
Kant defines reality as .Reality, in the pure conception of the understanding, is that which corresponds to a sensation in general . From that definition, insofar as only from the senses, and correspondingly by the sensations given from them, is any account of reality possible. This just says reality is given to us if or when the senses deliver sensations. So it is that the senses are in fact involved in both the external (input: effect of that thing which appears) and the internal (output: as affect corresponding to the appearance, which just is sensation). A completely legitimate explanatory bridge.
I probably should have just said the senses allow us to distinguish. Or, allow a distinction to be possible.
-
Quoting Metaphysician Undercover
Might help to know what the ever-so-abstract logical hole actually is, where it resides, and the complications arising from it. Knowing that, it becomes clear there is, not a contradiction but a theoretical inconsistency, inherent in noumena. It is not itself a self-contradictory idea, but it is an unintelligible object.
And Kant doesnt, indeed cannot, deny the possibility of noumena, insofar as to do so is to falsify the primary ground of transcendental philosophy, re: .I can think whatever I please, provided only that I do not contradict myself , which just says if I do think noumena, which is to hold a certain conception, and then prescribe to myself an object corresponding to it, then I immediately contradict the mechanisms I already authorize as that by which corresponding objects are prescribed to me at all, from which follows I have contradicted myself. The warning ends up being. think noumena all you like; just dont try to do anything intuitive with it. And if you cant do anything intuitive with it, dont bother thinking it in the first place.
The logical proof, and thereby the unintelligibility, is in the mechanism by which objects are prescribed on the one hand, which is determined by the very specific functionality of individual faculties on the other.
The legitimizing of noumena resides in a cognitive system I do not possess, arising for no other reason than I cannot say the cognitive system I possess is the only one there is. Phenomena belong to humans, noumena might belong to dolphins, or honey bees, or some rationality unknown to us. Which is .DUH!!!! ..all of them.
On what basis did Aristotle designate man the rational animal?
The faculty of reason is a perfectly intelligible expression, and the idea that humans alone possess it fully developed, and some animals only in very rudimentary forms, ought hardly need to be stated. Yet for some reason whenever it is stated, it provokes a good deal of argument. Which I attribute to the irrationality of modern culture!
I am familiar with this common argument and it has always left me somewhat cold. I don't have anything devastating against this view just some random thoughts. And yes, I'll be using reason.
Your wording seems very biased when you write things like "fully developed" and "very rudimentary forms" Surely that's a contingent viewpoint based on a series of assumptions?
This view is entirely predicated on us identifying ourselves as special - humans seem to have an innate ability to determine that we are favoured creatures of gods, and better/smarter than everything else on the planet. Is this not also one of our great blind spots - putting ourselves at the centre? Our reasoning is often indistinguishable from monomania. Perhaps this is why we have worked very hard to destroy the world and its wildlife. Reasoning often takes us to oblivion.
Is the line between us and animals so special because we have atom bombs and iPhones? Are our more complex adaptations and affectations a sign of superiority or really a kind of deficit?
It might even be argued that our particular brand of reasoning makes us inferior to animals who have and can find and do everything they need much more simply and elegantly than humans. They need no internet, no space programs, no Vogue magazine, schools or social media to thrive and live in harmony with nature. I'm not convinced that complexity equals superiority.
Our reasoning produces some useful and remarkable things (to us), but much reasoning is weak and bias ridden, and poorly inferred and dependent upon heuristics and simplifications. Humans have epic limitations on using reason which suggest we are simple and confused. (Yes, I know, this is your cue for something about higher actualization.)
Isn't one of the key arguments in Evan Thompson's Mind in Life: Biology, Phenomenology, and the Sciences of Mind (2007) that consciousness is enactive? That is, it arises from dynamic interactions between the body and the world rather than being an intrinsic property or essentialist trait? This isnt my area, but that sounds fascinating and I wonder what this says about animals.
Im going to make a spinoff thread and respond, but it will tomorrow.
Quoting Tom Storm
That is how Christians are said to have construed it, which fact is then regarded as an argument against it. But the Greeks proclaimed the sovereignty of reason before Jesus came along. Again, any argument against it must appeal to the very faculty which it seeks to question. It must give reasons.
Quoting Tom Storm
A moral judgement which no animal would make. There is nothing better or worse for them. Theyre not able to envisage that things could be otherwise than what they are.
Quoting Ludwig V
The issue is with the proposed analogous term, "window". The term characterizes the senses in a descriptive way. So we can ask, is the description accurate? Suppose the senses are like a window, we can apply the tinted glass analogy, and ask how is the window itself affecting our perception of what's on the other side. And when we look at the reality of being alive, we see that life is active, and then we need to allow that the supposed "window" is not a passive pane of glass, but it is actively doing something.
And if we say that the activity of the mind is to interpret the information, then why wouldn't we say that the activity of the sense organ itself is an interpretation? So the information received to the conscious mind, from the sense organ, is already a type of interpretation. And if we assume that the rational mind is a distinct and higher faculty (a difference of kind, as @Wayfarerdoes), then we ought to accept that the interpretations given to the rational mind by the senses are less reliable.
Quoting Mww
OK, you say that intuition provides the boundary between the senses as out there, and the appearances in the mind, as in here. But I think this produces the problem of determining where exactly the sensation is, in here, or out there. Kant, I believe, obscures the problem by talking about "sensibility", implying the potential for, or possibility of sensation, rather than talking about actual sensation. So "the faculty of intuition", may in this way, provide the mind (the internal) with the capacity to be receptive to sense activity, but this only veils the underlying problem of where exactly the sensation is, in here (in the mind), or out there (in the sense organ).
Quoting Mww
It sure seems to me like sensations are in the senses. When I touch something and feel its texture, warmth, softness, etc., I feel these sensations right in my fingertips. I taste things right in my mouth, and smell things in my nose. Sounds appear to be right inside my ears, and visual images appear to be in my eyes. All of my sensations appear to me to be right in the organs which sense them.
Quoting Mww
"Heavy" is not properly a sensation, but if we considered it as a sensation, we would feel it in the muscles, the pain in the muscles which are lifting the weight. The pain (sensation) is felt to be right at the location where it is sensed. Likewise, sound is heard to be right in the ears. You say that ears register vibrations, but you are not talking about "sensation" any more. If we stick to sensation, we need to recognize that the sensation we call sound, appears to be right within the ears.
Furthermore, we need to acknowledge that we are talking about appearances, and the appearance is that the sensation is right in the sense organ. This is how the concept of "intuition" clouds and obscures the issue. Instead of acknowledging that the sense organ is part of "in here", because the sensation is in the sense organ, "intuition" produces a boundary. Then "sensibility", as the capacity to sense gets placed on the other side of the boundary from the sense organ, instead of putting the capacity to sense within the organ itself. And this classifies all the senses together, as activity in the same category of "sensibility".
However, we really need to look at each different sense as possibly a distinct capacity, with a distinct object. If they are distinct, then there is a difference of kind between them, and it's wrong to look at sensibility as one capacity of the mind, provided by "intuition". We need to look at each sense as a distinct capacity, afforded by the corresponding sense organ, and determine what provides the possibility for each one.
Quoting Mww
The point though, is that the concepts of "intuition" and "sensibility" lead to an unsound description of sensation. So there is an unnecessary, and I would say unjustifiable boundary created between the mind, as the unity of the faculties, and the sense organs which are left outside the mind, as other than faculties.
Quoting Mww
I wouldn't say the gap is bridged legitimately. You have conveniently left out the role of intuition here, to create the appearance of legitimacy. If we include "intuition", then we see that sensations are not delivered by the senses, they are delivered by intuition, as providing sensibility, the possibility for sensation. Then the gap remains, but it is between intuition and the senses, and there is no necessity to have "the senses deliver sensations". Intuition could deliver sensations (as in dreams), without the role of the senses at all. And so the senses are not involved in the internal at all.
So the imposition of the concept "intuition" produces the appearance that the gap has been bridged with that intermediary, intuition. However, in reality the proposed medium "intuition" is placed completely on one side of the gap, and therefore does not actually provide a bridge.
Quoting Mww
"Unintelligible object" is contradictory by traditional Aristotelian principles. An object necessarily has a form, as its identity, and "form" is intelligible. If it has no identity, form, it cannot be said to be an object. Notice that "intelligible" signifies the possibility of being grasped by an intellect, so actually being apprehended by a human intellect is not required. This is how Aristotle excluded "prime matter" (matter without form) from reality, by showing that it cannot be an external, independent object, it can only be an idea in the mind. But as an idea in the mind it is self-contradicting, therefore it is excluded from the mind as well.
This is why the Christian metaphysicians assigned "Form" as the necessary aspect of the independent objects, and "matter" accounts for the accidents and contingencies, observed within the independent objects. The "logical hole" is then filled with an intelligible Form, God, which is to the highest degree intelligible, yet cannot be apprehended by the human intellect due to the human intellect's dependence on matter. The hole is filled because God is intelligible, yet unintelligible to humans, so we do not have self-contradiction within that form, or idea.
Quoting Mww
This is how Kant turns things around from the traditional Christian perspective. The tradition holds that the boundary, which is "the unintelligible", is matter, and places that as external to the mind. Kant brings the boundary as "intuition" into the mind. This allows "noumena" as a sort of replacement to "matter", being the unintelligible, to creep into the mind, in that area of the mind portioned off by the boundary, intuition. The self-contradictory concept is hidden behind intuition, so to speak. Within the mind it is nothing but a useless self-contradicting concept, unintuitive. Therefore we can see the need to reject it completely. But this requires restructuring the boundary so as not to allow it in there, behind the intuitions.
Quoting Wayfarer
The problem I see here, is that we, in our rational thinking impose strict boundaries on species of life forms. These boundaries are produced for epistemic purposes, although they may be based in real ontological principles, such as reproductive capacity. So we tend to believe that the differences indicated by our divisions of species, are real differences of kind even though they are really just established for epistemic purposes.
The real problem which this creates is an inability to adequately understand evolution. Once we set up those differences of kind between the species, we rob ourselves of the conceptual tools required to properly understand the interspecies relations which are essential to that process called evolution. So the divisions into types are very useful for some purposes, but as Plato shows in The Sophist those purposes could include sophistry. This means that we need to call into question, to doubt the boundaries which are drawn, because they are generally drawn for a purpose which might not be representative of good metaphysics.
So for example, you say some animals possess the faculty of reason "in very rudimentary forms". Isn't this indicative to you, that this difference is a difference of degree? I think, that to assign to human beings a distinct faculty, which is a difference of kind, we need to exclude these rudimentary forms from the others. And I am not at all arguing that this is impossible. As I said, I think that some of the distinct senses we have demonstrate differences of kind. But this creates a further problem which is the need to understand how differences of kind can evolve.
This is an ancient problem, traditionally resolved by having God create the different kinds of life forms individually. Denying the reality of differences of kind altogether, allows for the reality of evolution, and emergence, but this produces a different problem, the one encountered by the ancient atomists. To avoid an infinite regress of divisibility to account for all the differences, we need to assume a first basic, universal kind, a fundamental particle, known as prime matter. But as Aristotle showed, "prime matter" is actually incoherent. This implies that differences of kind must be real, and ontological, but we just do not have the adequate principles of understanding to properly identify them. And so we continue to assign "difference of kind" by somewhat arbitrary principles, according to the purpose at hand.
Quoting Metaphysician Undercover
Ive already stated that appearance is that which is an effect on the senses, as input; appearances, then, and the senses can be said to be out there. How the senses react to that which appears, as output, is sensation, which can be said to be in here. There is nothing to be gained procedurally speaking, by asking for a boundary, when all thats necessary is a transformation of whatever kind, between out there and in here. Ive also said already the human is not conscious of all that transpires between the appearance of a thing to the senses, and the judgement attributed to it, and if were not conscious of it we can neglect the effort required for determining wherever some boundary may be.
Quoting Metaphysician Undercover
And thats all we need to move on to the next faculty, the next procedural step on the way to determining how the appearance is to be known. There is an explanation for what intuition does pursuant to speculative metaphysics, but, again, the subject himself, being unconscious of the what, has even less need of the how.
-
Quoting Metaphysician Undercover
Thats fine, if you like. Ill stay with the effect of things on my sensory devices on the one hand, and the sensations such effects provide, accommodated by the type of sensory devices I possess, on the other. All I need is an input to the faculty of intuition, something from which phenomenon can be constructed. This is required in order to determine which sense has been affected, and what
a posteriori material is being processed, in which form may be imagined as belonging to it, and, VOILA!!! a very basic image is born.
-
Quoting Metaphysician Undercover
Not left out; yet to be a systemic consideration; intuition plays no role in perception, but only in the formulae for representing that which is perceived.
-
Quoting Metaphysician Undercover
Agreed. And where, in Kantian transcendental metaphysics, does form reside?
Quoting Metaphysician Undercover
Intelligible means necessarily cognizable by the human intellect, re: all logical criteria have been met. Unintelligible, then, merely means a cognition is impossible, even if a representation relating to a conception, is not. So what makes a conception a legitimate thought, but for which schemata representing it, is not at all possible? Whats missing?
-
Quoting Metaphysician Undercover
Interesting approach, but very far from the textual explanation.
Anyway ..enough for now.
Of course there is something to be gained here, because if we assume a boundary, then knowledge would be gained by understanding the nature of that boundary. If it is a "transformation" then there must be a cause of it, and understanding the source of that cause would be very important. Even the very existence of this supposed "transformation" casts doubt on the accuracy of our intuitions of space and time, as the transformation would be prior to the a priori. It would be the condition for the condition of sensibility.
In other words, we'd have to assume an active, animated cause of sensation, the transformation occurring at that boundary, which is prior to the a priori intuitions, and this would render those tools, (the intuitions) as useless for understanding reality. We would need to obtain a full understanding of that transformation, as the a priori intuitions are already posterior to that transformation, being applied to the results of the transformation. This produces the requirement for a huge procedural difference if one's goal is to understand reality.
.Quoting Mww
But if the named "subject" is a metaphysician, then by the nature of a metaphysician, the subject has a need of the how. That is exactly what the metaphysician wants to know. And if it turns out that the a priori intuitions are already posterior to the transformation which occurs at the senses, then the conceptual structure is useless to the metaphysician because it places that transformation into the category of noumenal. This can only leave the metaphysician dissatisfied by what appears to be a faulty procedure.
Quoting Mww
The problem though, is that this "input" is already the product of an active, animated process, which you call "a transformation of whatever kind", that has occurred at the boundary between out there and in here. So the "basic image" is merely a representation of the product of that transformation. Therefore if we want to understand the true nature of the "out there", we need to recognize that the intuitions of space and time are not being applied to the "out there", they are being applied to the product of the transformation, which is already "in here". So we need to get beyond these intuitions of space and time, and understand the nature of that transformation, if we want a true understanding of the "out there".
Quoting Mww
This, I think is a key point. The philosophical mind seeks to know everything, so it is counterproductive, sort of hypocritical, to designate anything as unintelligible. So the Aristotelian approach is to designate that to be an object is to have a form, therefore to be intelligible, as the law of identity indicates. However, Aristotle also noted the reality of "potential", as a sort of possibility for an object. And "potential" violates the law of excluded middle, throwing the intelligibility of that aspect of reality into doubt. This possibility of an unintelligible aspect of reality is not pleasing to the philosophical mind.
The metaphysician has to be crafty to avoid being stymied by the prospect of an unintelligible aspect of reality, so the traditional solution is to assume the reality of an intellect with a higher capacity than the capacity of the current human intellect, that would be God. To avoid the connotations of that name, we might call this a potential intellect, a possible intellect, which could understand the reality of things which appear to be unintelligible to the human intellect. This allows us to place into the category of "intelligible" things like independent Forms, which provide for the intelligibility of the reality of potential, and which the human intellect cannot grasp, making them appear as unintelligible potential. By this little trick, your definition of "intelligible" as "necessarily cognizable by the human intellect" is rejected as unacceptable.
Furthermore, the "trick" is well supported by evolutionary evidence. Living beings are extremely varied, and they evolve to develop different capacities. So the idea of an intellect (possibly a future more evolved intellect) which can grasp things which are unintelligible to the human intellect is clearly justified. And the so-called "trick" (which was formerly known as assuming God) is really a valid way for the metaphysician to get beyond the appearance of unintelligibility, and bring what appears to be unintelligible into the category of intelligible.